Preview (15 of 98 pages)

HESI EXIT RN EXAM V1-V7 2023
Version 1
Part 1
1. Following discharge teaching, a male client with duodenal ulcer tells the nurse the he will
drink plenty of dairy products, such as milk, to help coat and protect his ulcer. What is the
best follow-up action by the nurse?
Answer: Review with the client the need to avoid foods that are rich in milk and cream
2. A male client with hypertension, who received new antihypertensive prescriptions at his
last visit returns to the clinic two weeks later to evaluate his blood pressure (BP). His BP is
158/106 and he admits that he has not been taking the prescribed medication because the
drugs make him “feel bad”. In explaining the need for hypertension control, the nurse should
stress that an elevated BP places the client at risk for which pathophysiological condition?
Answer: Stroke secondary to haemorrhage
3. The nurse observes an unlicensed assistive personnel (UAP) positioning a newly admitted
client who has a seizure disorder. The client is supine and the UAP is placing soft pillows
along the side rails. What action should the nurse implement?
Answer: Instruct the UAP to obtain soft blankets to secure to the side rails instead of pillows.
4. An adolescent with major depressive disorder has been taking duloxetine (Cymbalta) for
the past 12 days. Which assessment finding requires immediate follow-up?
Answer: Describes life without purpose
5. A 60-year-old female client with a positive family history of ovarian cancer has developed
an abdominal mass and is being evaluated for possible ovarian cancer. Her Papanicolau (Pap)
smear results are negative. What information should the nurse include in the client’s teaching
plan?
Answer: Further evaluation involving surgery may be needed
6. A client who recently underwear a tracheostomy is being prepared for discharge to home.
Which instructions is most important for the nurse to include in the discharge plan?
Answer: Teach tracheal suctioning techniques

7. In assessing an adult client with a partial rebreather mask, the nurse notes that the oxygen
reservoir bag does not deflate completely during inspiration and the client’s respiratory rate is
14 breaths / minute. What action should the nurse implement?
Answer: Document the assessment data
Rationale:
Reservoir bag should not deflate completely during inspiration and the client’s respiratory
rate is within normal limits.
8. During shift report, the central electrocardiogram (EKG) monitoring system alarms.
Which client alarm should the nurse investigate firs?
Answer: Respiratory apnea of 30 seconds
9. During a home visit, the nurse observed an elderly client with diabetes slip and fall. What
action should the nurse take first?
Answer: Check the client for lacerations or fractures
10. At 0600 while admitting a woman for a schedule repeat cesarean section (C-Section), the
client tells the nurse that she drank a cup a coffee at 0400 because she wanted to avoid getting
a headache. Which action should the nurse take first?
Answer: Inform the anaesthesia care provider
11. After placing a stethoscope as seen in the picture, the nurse auscultates S1 and S2 heart
sounds. To determine if an S3 heart sound is present, what action should the nurse take first?
Answer: Listen with the bell at the same location
12. A 66-year-old woman is retiring and will no longer have a health insurance through her
place of employment. Which agency should the client be referred to by the employee health
nurse for health insurance needs?
Answer: Medicare
13. A client who is taking an oral dose of a tetracycline complains of gastrointestinal upset.
What snack should the nurse instruct the client to take with the tetracycline?
Answer: Toasted wheat bread and jelly

14. Following a lumbar puncture, a client voices several complaints. What complaint
indicated to the nurse that the client is experiencing a complication?
A. “I have a headache that gets worse when I sit up”
B. “I am having pain in my lower back when I move my legs”
C. “My throat hurts when I swallow”
D. “I feel sick to my stomach and am going to throw up”
Answer: A. “I have a headache that gets worse when I sit up”
15. An elderly client seems confused and reports the onset of nausea, dysuria, and urgency
with incontinence. Which action should the nurse implement?
Answer: Obtain a clean catch mid-stream specimen
16. The nurse is assisting the mother of a child with phenylketonuria (PKU) to select foods
that are in keeping with the child’s dietary restrictions. Which foods are contraindicated for
this child?
Answer: Foods sweetened with aspartame
17. Before preparing a client for the first surgical case of the day, a part-time scrub nurse asks
the circulating nurse if a 3 minute surgical hand scrub is adequate preparation for this client.
Which response should the circulating nurse provide?
Answer: Direct the nurse to continue the surgical hand scrub for a 5 minute duration
18. Which breakfast selection indicates that the client understands the nurse’s instructions
about the dietary management of osteoporosis?
Answer: Bagel with jelly and skim milk
19. The charge nurse of a critical care unit is informed at the beginning of the shift that less
than the optimal number of registered nurses will be working that shift. In planning
assignments, which client should receive the most care hours by a registered nurse (RN)?
Answer: An 82-year-old client with Alzheimer’s disease newly-fractures femur who has a
Foley catheter and soft wrist restrains applied

20. A mother brings her 6-year-old child, who has just stepped on a rusty nail, to the
paediatrician’s office. Upon inspection, the nurse notes that the nail went through the shoe
and pierced the bottom of the child’s foot. Which action should the nurse implement first?
A. Cleanse the foot with soap and water and apply an antibiotic ointment
B. Provide teaching about the need for a tetanus booster within the next 72 hours.
C. have the mother check the child's temperature q4h for the next 24 hours
D. transfer the child to the emergency department to receive a gamma globulin injection
Answer: A. Cleanse the foot with soap and water and apply an antibiotic ointment
21. The mother of an adolescent tells the clinic nurse, “My son has athlete’s foot, I have been
applying triple antibiotic ointment for two days, but there has been no improvement.” What
instruction should the nurse provide?
Answer: Stop using the ointment and encourage complete drying of the feet and wearing
clean socks.
22. A 26-year-old female client is admitted to the hospital for treatment of a simple goiter,
and levothyroxine sodium (Synthroid) is prescribed. Which symptoms indicate to the nurse
that the prescribed dosage is too high for this client? The client experiences
A. Bradycardia and constipation
B. Lethargy and lack of appetite
C. Muscle cramping and dry, flushed skin
D. Palpitations and shortness of breath
Answer: D. Palpitations and shortness of breath
23. A client with a history of heart failure presents to the clinic with a nausea, vomiting,
yellow vision and palpitations. Which finding is most important for the nurse to assess to the
client?
Answer: Obtain a list of medications taken for cardiac history
24. The healthcare provider prescribes an IV solution of isoproterenol (Isuprel) 1 mg in 250
ml of D5W at 300 mcg/hour. The nurse should program the infusion pump to deliver how
many ml/hour? (Enter numeric value only.)
Answer: 75
Rationale:

Convert mg to mcg and use the formula D/H x Q. 300 mcg/hour / 1,000 mcg x 250 ml = 3/1 x
25 = 75 ml/hour
25. The pathophysiological mechanism are responsible for ascites related to liver failure?
(Select all that apply)
A. Fluid shifts from intravascular to interstitial area due to decreased serum protein
B. Increased hydrostatic pressure in portal circulation increases fluid shifts into abdomen
C. Increased circulating aldosterone levels that increase sodium and water retention
Answer: A. Fluid shifts from intravascular to interstitial area due to decreased serum protein
B. Increased hydrostatic pressure in portal circulation increases fluid shifts into abdomen
C. Increased circulating aldosterone levels that increase sodium and water retention
26. The nurse is auscultating a client’s heart sounds. Which description should the nurse use
to document this sound? (Please listen to the audio first to select the option that applies)
Answer: Murmur
Rationale:
A murmur is auscultated as a swishing sound that is associated with the blood turbulence
created by the heart or valvular defect.
27. The healthcare provider prescribes celtazidime (Fortax) 35 mg every 8 hours IM for an
infant. The 500 mg vial is labelled with the instruction to add 5.3 ml diluent to provide a
concentration of 100 mg/ml. How many ml should the nurse administered for each dose?
(Enter numeric value only. If rounding is required, round to the nearest tenth)
Answer: 0.4
Rationale:
35mg/100mg x 1 = 0.35 = 0.4 ml
28. The nurse notes that a client has been receiving hydromorphone (Dilaudid) every six
hours for four days. What assessment is most important for the nurse to complete?
A. Auscultate the client's bowel sounds
B. Observe for edema around the ankles
C. Measure the client’s capillary glucose level
D. Count the apical and radial pulses simultaneously
Answer: A. Auscultate the client's bowel sounds

Rationale:
Hydromorphone is a potent opioid analgesic that slows peristalsis and frequently causes
constipation, so it is most important to Auscultate the client's bowel sounds
29. A female client is admitted with end stage pulmonary disease is alert, oriented, and
complaining of shortness of breath. The client tells the nurse that she wants “no heroic
measures” taken if she stops breathing, and she asks the nurse to document this in her
medical record. What action should the nurse implement?
Answer: Ask the client to discuss “do not resuscitate” with her healthcare provider
30. A client is receiving a full strength continuous enteral tube feeding at 50 ml/hour and has
developed diarrhoea. The client has a new prescription to change the feeding to half strength.
What intervention should the nurse implement?
Answer: Add equal amounts of water and feeding to a feeding bag and infuse at 50ml/hour
31. A female client reports that her hair is becoming coarse and breaking off, that the outer
part of her eyebrows have disappeared, and that her eyes are all puffy. Which follow-up
question is best for the nurse to ask?
Answer: Have you noticed any changes in your fingernails?
Rationale:
The pattern of reported manifestations is suggestive of hypothyroidism
32. After a third hospitalization 6 months ago, a client is admitted to the hospital with ascites
and malnutrition. The client is drowsy but responding to verbal stimuli and reports recently
spitting up blood. What assessment finding warrants immediate intervention by the nurse?
A. Capillary refill of 8 seconds
B. bruises on arms and legs
C. round and tight abdomen
D. pitting edema in lower legs
Answer: A. Capillary refill of 8 seconds
33. After the nurse witnesses a preoperative client sign the surgical consent form, the nurse
signs the form as a witness. What are the legal implications of the nurse’s signature on the
client’s surgical consent form? (Select all that apply)

A. The client voluntarily grants permission for the procedure to be done
B. The client is competent to sign the consent without impairment of judgment
C. The client understands the risks and benefits associated with the procedure
Answer: A. The client voluntarily grants permission for the procedure to be done
B. The client is competent to sign the consent without impairment of judgment
C. The client understands the risks and benefits associated with the procedure
34. Following surgery, a male client with antisocial personality disorder frequently requests
that a specific nurse be assigned to his care and is belligerent when another nurse is assigned.
What action should the charge nurse implement?
Answer: Advise the client that assignments are not based on clients requests
35. A client with cervical cancer is hospitalized for insertion of a sealed internal cervical
radiation implant. While providing care, the nurse finds the radiation implant in the bed.
What action should the nurse take?
Answer: Place the implant in a lead container using long-handled forceps
36. The client with which type of wound is most likely to need immediate intervention by the
nurse?
A. Laceration
B. Abrasion
C. Contusion
D. Ulceration
Answer: A. Laceration
Rationale:
A laceration is a wound that is produced by the tearing of soft body tissue. This type of
wound is often irregular and jagged. A laceration wound is often contaminated with bacteria
and debris from whatever object caused the cut.
37. The nurse is planning care for a client admitted with a diagnosis of pheochromocytoma.
Which intervention has the highest priority for inclusion in this client’s plan of care?
Answer: Monitor blood pressure frequently
Rationale:

A pheochromocytoma is a rare, catecholamine-secreting tumour that may precipitate lifethreatening hypertension. The tumour is malignant in 10% of cases but may be cured
completely by surgical removal. Although pheochromocytoma has classically been associated
with 3 syndromes—von Hippel-Lindau (VHL) syndrome, multiple endocrine neoplasia type
2 (MEN 2), and neurofibromatosis type 1 (NF1)—there are now 10 genes that have been
identified as sites of mutations leading to pheochromocytoma.
38. When caring for a client who has acute respiratory distress syndrome (ARDS), the nurse
elevates the head of the bed 30 degrees. What is the reason for this intervention?
A. To reduce abdominal pressure on the diaphragm
B. to promote retraction of the intercostal accessory muscle of respiration
C. to promote bronchodilation and effective airway clearance
D. to decrease pressure on the medullary centre which stimulates breathing
Answer: A. To reduce abdominal pressure on the diaphragm
Rationale:
A semi-sitting position is the best position for matching ventilation and perfusion and for
decreasing abdominal pressure on the diaphragm, so that the client can maximize breathing.
39. When assessing a mildly obese 35-year-old female client, the nurse is unable to locate the
gallbladder when palpating below the liver margin at the lateral border of the rectus
abdominal muscle. What is the most likely explanation for failure to locate the gallbladder by
palpation?
A. The client is too obese
B. Palpating in the wrong abdominal quadrant
C. Deeper palpation technique is needed
D. The gallbladder is normal
Answer: D. The gallbladder is normal
Rationale:
A normal healthy gallbladder is not palpable
40. A woman with an anxiety disorder calls her obstetrician’s office and tells the nurse of
increased anxiety since the normal vaginal delivery of her son three weeks ago. Since she is
breastfeeding, she stopped taking her antianxiety medications, but thinks she may need to

start taking them again because of her increased anxiety. What response is best for the nurse
to provide this woman?
A. describe the transmission of drugs to the infant through breast milk
B. encourage her to use stress relieving alternatives, such as deep breathing exercises
C. Inform her that some antianxiety medications are safe to take while breastfeeding
D. Explain that anxiety is a normal response for the mother of a 3-week-old.
Answer: C. Inform her that some antianxiety medications are safe to take while breastfeeding
Rationale:
There are several antianxiety medications that are not contraindicated for breastfeeding
mothers.
41. An older male client with a history of type 1 diabetes has not felt well the past few days
and arrives at the clinic with abdominal cramping and vomiting. He is lethargic, moderately,
confused, and cannot remember when he took his last dose of insulin or ate last. What action
should the nurse implement first?
A. Start an intravenous (IV) infusion of normal saline
B. obtain a serum potassium level
C. administer the client's usual dose of insulin
D. assess pupillary response to light
Answer: A. Start an intravenous (IV) infusion of normal saline
Rationale:
The nurse should first start an intravenous infusion of normal saline to replace the fluids and
electrolytes because the client has been vomiting, and it is unclear when he last ate or took
insulin. The symptoms of confusion, lethargy, vomiting, and abdominal cramping are all
suggestive of hyperglycaemia, which also contributes to diuresis and fluid electrolyte
imbalance.
42. A client who received multiple antihypertensive medications experiences syncope due to
a drop in blood pressure to 70/40. What is the rationale for the nurse’s decision to hold the
client’s scheduled antihypertensive medication?
A. increased urinary clearance of the multiple medications has produced diuresis and lowered
the blood pressure
B. the antagonistic interaction among the various blood pressure medications has reduced
their effectiveness

C. The additive effect of multiple medications has caused the blood pressure to drop too low
D. the synergistic effect of the multiple medications has resulted in drug toxicity and resulting
hypotension
Answer: C. The additive effect of multiple medications has caused the blood pressure to drop
too low
43. Which client is at the greatest risk for developing delirium?
A. An adult client who cannot sleep due to constant pain.
B. an older client who attempted 1 month ago
C. a young adult who takes antipsychotic medications twice a day
D. a middle-aged woman who uses a tank for supplemental oxygen
Answer: A. An adult client who cannot sleep due to constant pain.
44. Which intervention should the nurse include in a long-term plan of care for a client with
Chronic Obstructive Pulmonary Disease (COPD)?
A. Reduce risks factors for infection
B. Administer high flow oxygen during sleep
C. Limit fluid intake to reduce secretions
D. Use diaphragmatic breathing to achieve better exhalation
Answer: A. Reduce risks factors for infection
45. Which location should the nurse choose as the best for beginning a screening program for
hypothyroidism?
A. A business and professional women's group.
B. An African-American senior citizens centre
C. A daycare centre in a Hispanic neighbourhood
D. An after-school centre for Native-American teens
Answer: A. A business and professional women's group.
46. A female client has been taking a high dose of prednisone, a corticosteroid, for several
months. After stopping the medication abruptly, the client reports feeling “very tired”. Which
nursing intervention is most important for the nurse to implement?
A. Measure vital signs
B. Auscultate breath sounds

C. Palpate the abdomen
D. Observe the skin for bruising
Answer: A. Measure vital signs
47. A male client reports the onset of numbness and tingling in his fingers and around his
mouth. Which lab is important for the nurse to review before contacting the health care
provider?
A. capillary glucose
B. urine specific gravity
C. Serum calcium
D. white blood cell count
Answer: C. Serum calcium
48. What explanation is best for the nurse to provide a client who asks the purpose of using
the log-rolling technique for turning?
A. working together can decrease the risk for back injury
B. The technique is intended to maintain straight spinal alignment.
C. Using two or three people increases client safety.
D. turning instead of pulling reduces the likelihood of skin damage
Answer: B. The technique is intended to maintain straight spinal alignment.
49. A client receiving chemotherapy has severe neutropenia. Which snack is best for the nurse
to recommend to the client?
Answer: Baked apples topped with dried raisins
50. Which action should the school nurse take first when conducting a screening for
scoliosis?
Answer: Inspect for symmetrical shoulder height.
51. An unlicensed assistive personnel (UAP) assigned to obtain client vital signs reports to
the charge nurse that a client has a weak pulse with a rate of 44 beat/ minutes. What action
should the charge nurse implement?
Answer: Assign a practical nurse (LPN) to determine if an apical radial deficit is present

52. After a sudden loss of consciousness, a female client is taken to the ED and initial
assessment indicate that her blood glucose level is critically low. Once her glucose level is
stabilized, the client reports that was recently diagnosed with anorexia nervosa and is being
treated at an outpatient clinic. Which intervention is more important to include in this client’s
discharge plan?
Answer: Encourage a low-carbohydrate and high-protein diet
53. A client with a peripherally inserted central catheter (PICC) line has a fever. What client
assessment is most important for the nurse to perform?
Answer: Observe the antecubital fossa for inflammation.
54. The nurse administers an antibiotic to a client with respiratory tract infection. To evaluate
the medication’s effectiveness, which laboratory values should the nurse monitor? Select all
that apply
A. White blood cell (WBC) count
B. Sputum culture and sensitivity
Answer: A. White blood cell (WBC) count
B. Sputum culture and sensitivity
55. A client is admitted to isolation with the diagnosis of active tuberculosis. Which infection
control measures should the nurse implement?
A. Negative pressure environment
B. contact precautions
C. droplet precautions
D. protective environment
Answer: A. Negative pressure environment
56. A school nurse is called to the soccer field because a child has a nose bleed (epistaxis). In
what position should the nurse place the child?
Answer: Sitting up and leaning forward
57. A young adult who is hit with a baseball bat on the temporal area of the left skull is
conscious when admitted to the ED and is transferred to the Neurological Unit to be

monitored for signs of closed head injury. Which assessment finding is indicative of a
developing epidural hematoma?
Answer: Altered consciousness within the first 24 hours after injury.
58. A female client with breast cancer who completed her first chemotherapy treatment today
at an out-patient centre is preparing for discharge. Which behavior indicates that the client
understands her care needs
Answer: Rented movies and borrowed books to use while passing time at home
59. Which instruction should the nurse provide a pregnant client who is complaining of
heartburn?
Answer: Eat small meal throughout the day to avoid a full stomach.
60. A client is admitted to the intensive care unit with diabetes insipidus due to a pituitary
gland tumour. Which potential complication should the nurse monitor closely?
A. Hypokalaemia
B. Ketonuria.
C. Peripheral edema
D. Elevated blood pressure
Answer: A. Hypokalemia
Rationale:
Pituitary tumors that suppress antidiuretic hormone (ADH) result in diabetes insipidus, which
causes massive polyuria and serum electrolyte imbalances, including hypokalemia, which can
lead to lethal arrhythmias.
61. A female client reports she has not had a bowel movement for 3 days, but now is
defecating frequent small amount of liquid stool. Which action should the nurse implement?
Answer: Digitally check the client for a fecal impaction
62. After changing to a new brand of laundry detergent, an adult male reports that he has a
fine itchy rash. Which assessment finding warrants immediate intervention by the nurse?
Answer: Bilateral Wheezing.

63. The nurse should teach the parents of a 6 year-old recently diagnosed with asthma that the
symptom of acute episode of asthma are due to which physiological response?
Answer: Inflammation of the mucous membrane & bronchospasm
64. A 10 year old who has terminal brain cancer asks the nurse, "What will happen to my
body when I die?" How should the nurse respond?
Answer: "The heart will stop beating & you will stop breathing."
65. The nurse is assessing a 3-month-old infant who had a pylorotomy yesterday. This child
should be medicated for pain based on which findings? Select all that apply:
A. Restlessness
B. Clenched Fist
C. Increased pulse rate
D. Increased respiratory rate.
E. Increased temperature
F. Peripheral pallor of the skin
Answer: A. Restlessness
B. Clenched Fist
C. Increased pulse rate
D. Increased respiratory rate.
66. The nurse is preparing to administer an oral antibiotic to a client with unilateral weakness,
ptosis, mouth drooping and, aspiration pneumonia. What is the priority nursing assessment
that should be done before administering this medication?
Answer: Determine which side of the body is weak.
67. A Nurse who is working on a surgical unit receives change of shift report on a group of
clients for the upcoming shift. A client with which condition requires the most immediate
attention by the nurse?
A. Gunshot wound three hours ago with dark drainage of 2 cm noted on the dressing.
B. Mastectomy 2 days ago with 50 ml bloody drainage noted in the Jackson-pratt drain.
C. Collapsed lung after a fall 8h ago with 100 ml blood in the chest tube collection container
D. Abdominal-perineal resection 2 days ago with no drainage on dressing who has fever and
chills.

Answer: D. Abdominal-perineal resection 2 days ago with no drainage on dressing who has
fever and chills.
Rationale:
The client with an abdominal- perineal resection is at risk for peritonitis and needs to be
immediately assessed for other signs and symptoms for sepsis.
68. The nurse is caring for a client who had gastric bypass surgery yesterday. Which
intervention is most important for the nurse to implement during the first 24 postoperative
hours?
Answer: Measure hourly urinary output.
Rationale:
A serious early complications of gastric bypass surgery is an anastomoses leak, often
resulting in death.
69. When preparing to discharge a male client who has been hospitalized for an adrenal crisis,
the client expresses concern about having another crisis. He tells the nurse that he wants to
stay in the hospital a few more days. Which intervention should the nurse implement?
Answer: Schedule an appointment for an out-patient psychosocial assessment.
70. An adult female client tells the nurse that though she is afraid her abusive boyfriend might
one day kill her; she keeps hoping that he will change. What action should the nurse take
first?
Answer: Explore client’s readiness to discuss the situation.
71. In caring for a client with Cushing syndrome, which serum laboratory value is most
important for the nurse to monitor?
A. Lactate
B. Glucose
C. Haemoglobin
D. Creatinine
Answer: B. Glucose

72. Azithromycin is prescribed for an adolescent female who has lower lobe pneumonia and
recurrent chlamydia. What information is most important for the nurse to provide to this
client?
Answer: Use two forms of contraception while taking this drug.
73. A client in the emergency center demonstrates rapid speech, flight of ideas, and reports
sleeping only three hours during the past 48h. Based on these finding, it is most important for
the nurse to review the laboratory value for which medication?
Answer: Divalproex.
Rationale:
Divalproex is the first line of treatment for bipolar disorder BPD because it has a high
therapeutic index, few side effects, and a rapid onset in controlling symptoms and preventing
recurrent episodes of mania and depression. The serum value of divalproex should be
determined since the client is exhibiting symptoms of mania, which may indicate noncompliance with the medication regimen.
74. A male client who is admitted to the mental health unit for treatment of bipolar disorder
has a slightly slurred speech pattern and an unsteady gait. Which assessment finding is most
important for the nurse to report to the healthcare provider?
Answer: Serum lithium level of 1.6 mEq/L or mmol/l (SI)
Rationale:
The therapeutic level of Serum lithium is 0.8 to 1.5 mEq/L or mmol/l (SI). Slurred speech and
ataxia are sign of lithium toxicity.
75. A client was admitted to the cardiac observation unit 2 hours ago complaining of chest
pain. On admission, the client’s EKG showed bradycardia, ST depression, but no ventricular
ectopy. The client suddenly reports a sharp increase in pain, telling the nurse, “I feel like an
elephant just stepped on my chest” The EKG now shows Q waves and ST segment elevations
in the anterior leads. What intervention should the nurse perform?
Answer: Administer prescribed morphine sulfate IV and provide oxygen at 2 L/min per nasal
cannula .
76. The nurse is developing a teaching program for the community. What population
characteristic is most influential when choosing strategies for implementing a teaching plan?

Answer: Literacy level
77. A client is being discharged with a prescription for warfarin (Coumadin). What instruction
should the nurse provide this client regarding diet?
Answer: Eat approximated the same amount of leafy green vegetables daily so the amount of
vitamin K consumed is consistent.
78. A client who had a small bowel resection acquired methicillin resistant staphylococcus
aureus (MRSA) while hospitalized. He treated and released, but is readmitted today because
of diarrhoea and dehydration. It is most important for the nurse to implement which
intervention.
Answer: Maintain contact transmission precaution
79. A postoperative female client has a prescription for morphine sulphate 10 mg IV q3 hours
for pain. One dose of morphine was administered when the client was admitted to the post
anaesthesia care unit (PACU) and 3 hours later, the client is again complaining of pain. Her
current respiratory rate is 8 breaths/minute. What action should the nurse take?
Answer: Administer Naloxone IV
80. Which intervention is most important for the nurse to include in the plan of care for an
older woman with osteoporosis?
Answer: Place the client on fall precautions
81. Based on the information provided in this client’s medical record during labor, which
should the nurse implement? (Click on each chart tab for additional information. Please be
sure to scroll to the bottom right corner of each tab to view all information contained in the
client’s medical record.)
Answer: Continue to monitor the progress of labor.
82. An unlicensed assistive personnel UAP leaves the unit without notifying the staff. In what
order should the unit manager implement this intervention to address the UAPs behavior?
(Place the action in order from first on top to last on bottom.)
1. Note date and time of the behavior.
2. Discuss the issue privately with the UAP.

3. Plan for scheduled break times.
4. Evaluate the UAP for signs of improvement.
Answer: 1. Note date and time of the behavior.
2. Discuss the issue privately with the UAP.
3. Plan for scheduled break times.
4. Evaluate the UAP for signs of improvement.
83. A client with intestinal obstructions has a nasogastric tube to low intermittent suction and
is receiving an IV of lactated ringer’s at 100 ml/H. which finding is most important for the
nurse to report to the healthcare provider?
Answer: Serum potassium level of 3.1 mEq/L or mmol/L (SI)
Rationale:
The normal potassium level in the blood is 3.5-5.0 milliEquivalents per liter (mEq/L).
84. Which type of Leukocyte is involved with allergic responses and the destruction of
parasitic worms?
A. Neutrophils
B. Lymphocytes
C. Eosinophils
D. Monocytes
Answer: C. Eosinophils
Rationale:
Eosinophils are involved in allergic responses and destruction of parasitic worms.
85. The healthcare provider prescribes the antibiotic cephradine 500mg PO every 6 hours for
a client with a postoperative wound infection. Which foods should the nurse encourage this
client to eat?
Answer: Yogurt and/or buttermilk.
86. Several months after a foot injury, and adult woman is diagnosed with neuropathic pain.
The client describes the pain as severe and burning and is unable to put weight on her foot.
She asks the nurse when the pain will “finally go away.” How should the nurse respond?
Answer: Assist the client in developing a goal of managing the pain

87. One day following an open reduction and internal fixation of a compound fracture of the
leg, a male client complains of “a tingly sensation” in his left foot. The nurse determines the
client’s left pedal pulses are diminished. Based on these finding, what is the client’s greatest
risk?
Answer: Neurovascular and circulation compromise related to compartment syndrome.
88. The nurse is completing a head to be assessment for a client admitted for observation
after falling out of a tree. Which finding warrants immediate intervention by the nurse?
Answer: Clear fluid leaking from the nose.
89. A client with multiple sclerosis (MS) has decreased motor function after taking a hot bath
(Uhthoff’s sign). Which pathophysiological mechanism supports this response?
Answer: Temporary vasodilation
90. While assessing a radial artery catheter, the client complains of numbness and pain distal
to the insertion site. What interventions should the nurse implement?
Answer: Promptly remove the arterial catheter from the radial artery.
91. A client is admitted with an epidural hematoma that resulted from a skateboarding
accident. To differentiate the vascular source of the intracranial bleeding, which finding
should the nurse monitor?
Answer: Rapid onset of decreased level of consciousness.
92. The nurse finds a client at 33 weeks gestation in cardiac arrest. What adaptation to
cardiopulmonary resuscitation (CPR) should the nurse implement?
Answer: Position a firm wedge to support pelvis and thorax at 30 degree tilt.
93. When preparing a client for discharge from the hospital following a cystectomy and a
urinary diversion to treat bladder cancer, which instruction is most important for the nurse to
include in the client’s discharge teaching plan?
Answer: Report any signs of cloudy urine output.

94. For the past 24 hours, an antidiarrheal agent, diphenoxylate, has been administered to a
bedridden, older client with infectious gastroenteritis. Which finding requires the nurse to
take further action?
Answer: Tented skin turgor.
95. After repositioning an immobile client, the nurse observes an area of hyperemia. To assess
for blanching, what action should the nurse take?
Answer: Apply light pressure over the area.
96. The nurse enters a client’s room and observes the client’s wrist restraint secured as seen in
the picture. What action should the nurse take?
Answer: Reposition the restraint tie onto the bedframe.
97. A female client with acute respiratory distress syndrome (ARDS) is chemically paralyzed
and sedated while she is on as assist-control ventilator using 50% FIO2. Which assessment
finding warrants immediate intervention by the nurse?
Answer: Diminished left lower lobe sounds
Rationale:
Diminished lobe sounds indicate collapsed alveoli or tension pneumothorax, which required
immediate chest tube insertion to re-inflate the lung.
98. The development of atherosclerosis is a process of sequential events. Arrange the
pathophysiological events in orders of occurrence. (Place the first event on top and the last on
the bottom)
1. Arterial endothelium injury causes inflammation
2. Macrophages consume low density lipoprotein (LDL), creating foam cells
3. Foam cells release growth factors for smooth muscle cells
4. Smooth muscle grows over fatty streaks creating fibrous plaques
5. Vessel narrowing results in ischemia
Answer: 1. Arterial endothelium injury causes inflammation
2. Macrophages consume low density lipoprotein (LDL), creating foam cells
3. Foam cells release growth factors for smooth muscle cells
4. Smooth muscle grows over fatty streaks creating fibrous plaques
5. Vessel narrowing results in ischemia

99. Following a motor vehicle collision, an adult female with a ruptured spleen and a blood
pressure of 70/44, had an emergency splenectomy. Twelve hours after the surgery, her urine
output is 25 ml/hour for the last two hours. What pathophysiological reason supports the
nurse’s decision to report this finding to the healthcare provider?
Answer: Oliguria signals tubular necrosis related to hypoperfusion
100. A nurse-manager is preparing the curricula for a class for charge nurses. A staffing
formula based on what data ensures quality client care and is most cost-effective?
Answer: Skills of staff and client acuity
101. When performing postural drainage on a client with Chronic Obstructive Pulmonary
Disease (COPD), which approach should the nurse use?
Answer: Explain that the client may be placed in five positions
102. A client presents in the emergency room with right-sided facial asymmetry. The nurse
asks the client to perform a series of movements that require use of the facial muscles. What
symptoms suggest that the client has most likely experience a Bell’s palsy rather than a
stroke?
Answer: Inability to close the affected eye, raise brow, or smile
103. The nurse is teaching a client how to perform colostomy irrigations. When observing the
client’s return demonstration, which action indicated that the client understood the teaching?
Answer: Keeps the irrigating container less than 18 inches above the stoma
104. The nurse should teach the client to observe which precaution while taking
dronedarone?
Answer: Avoid grapefruits and its juice
105. A client who sustained a head injury following an automobile collision is admitted to the
hospital. The nurse include the client’s risk for developing increased intracranial pressure
(ICP) in the plan of care. Which signs indicate to the nurse that ICP has increased?
A. Increased Glasgow coma scale score.
B. Nuchal rigidity and papilledema.

C. Confusion and papilledema
D. Periorbital ecchymosis.
Answer: C. Confusion and papilledema
Rationale:
Papilledema is always an indicator of increased ICP, and confusion is usually the first sign of
increased ICP. Other options do not necessarily reflect increased ICP.
106. The nurse is caring for a client receiving continuous IV fluids through a single lumen
central venous catheter (CVC). Based on the CVC care bundle, which action should be
completed daily to reduce the risk for infection?
Answer: Confirm the necessity for continued use of the CVC.
107. During an annual physical examination, an older woman’s fasting blood sugar (FBS) is
determined to be 140 mg/dl or 7.8 mmol/L (SI). Which additional finding obtained during a
follow-up visit 2 weeks later is most indicative that the client has diabetes mellitus (DM)?
Answer: Repeated fasting blood sugar (FBS) is 132 mg/dl or 7.4 mmol/L (SI).
108. A new mother tells the nurse that she is unsure if she will be able to transition into
parenthood. What action should the nurse take?
Answer: Determine if she can ask for support from family, friend, or the baby’s father.
109. A client who was admitted yesterday with severe dehydration is complaining of pain a
24 gauge IV with normal saline is infusing at a rate of 150 ml/hour. Which intervention
should the nurse implement first?
Answer: Stop the normal saline infusion.
110. An elderly female is admitted because of a change in her level of sensorium. During the
evening shift, the client attempts to get out bed and falls, breaking her left hip. Buck’s skin
traction is applied to the left leg while waiting for surgery. Which intervention is most
important for the nurse to include in this client’s plan care?
Answer: Ensure proper alignment of the leg in traction.

111. An Unna boot is applied to a client with a venous stasis ulcer. One week later, when the
Unna boot is removed during a follow-up appointment, the nurse observes that the ulcer site
contains bright red tissue. What action should the nurse take in response to this finding?
Answer: Document the ongoing wound healing.
112. At the end of a preoperative teaching session on pain management techniques, a client
starts to cry and states, “I just know I can’t handle all the pain.” What is the priority nursing
diagnosis for this client?
Answer: Anxiety
113. The nurse note a visible prolapse of the umbilical cord after a client experiences
spontaneous rupture of the membranes during labor. What intervention should the nurse
implement immediately?
Answer: Elevate the presenting part off the cord.
114. A client who had a right hip replacement 3 day ago is pale has diminished breath sound
over the left lower lung fields, a temperature of 100.2 F, and an oxygen saturation rate of
90%. The client is scheduled to be transferred to a skilled nursing facility (SNF) tomorrow
for rehabilitative critical pathway. Based on the client’s symptoms, what recommendation
should the nurse give the healthcare provider?
Answer: Reassess readiness for SNF transfer.
115. A client who is newly diagnosed with type 2 diabetes mellitus (DM) receives a
prescription for metformin (Glucophage) 500 mg PO twice daily. What information should
the nurse include in this client’s teaching plan? (Select all that apply.)
A. Recognize signs and symptoms of hypoglycemia.
B. Report persist polyuria to the healthcare provider.
C. Take Glucophage with the morning and evening meal.
Answer: A. Recognize signs and symptoms of hypoglycemia.
B. Report persist polyuria to the healthcare provider.
C. Take Glucophage with the morning and evening meal.

116. The nurse is developing an educational program for older clients who are being
discharged with new antihypertensive medications. The nurse should ensure that the
educational materials include which characteristics? Select all that apply
A. Written at a twelfth grade reading level
B. Contains a list with definitions of unfamiliar terms
C. Uses common words with few Syllables
D. Printed using a 12 point type font
E. Uses pictures to help illustrate complex ideas
Answer: B. Contains a list with definitions of unfamiliar terms
C. Uses common words with few Syllables
E. Uses pictures to help illustrate complex ideas
Rationale:
During the aging process older clients often experience sensory or cognitive changes, such as
decreased visual or hearing acuity, slower thought or reasoning processes, and shorter
attention span. Materials for this age group should include at least of terms, such as a medical
terminology that incline may not know and use common words that expresses information
clearly and simply. Simple, attractive pictures help hold the learner’s attention. The reading
level of material should be at the 4th to 5th grade level. Materials should be printed using
large font (18-point or higher), not the standard 12-point font.
117. During the admission assessment, the nurse auscultates heart sounds for a client with no
history of cardiovascular disease. Where should the nurse listen when assessing the client’s
point of maximal impulse (PMI) (Click the chosen location. To change, click on a new
location)
Answer:

118. An older male adult resident of long-term care facility is hospitalized for a cardiac
catheterization that occurred yesterday. Since the procedure was conducted, the client has
become increasingly disoriented. The night shift nurse reports that he attempted to remove
the sandbag from his femoral artery multiple times during the night. What actions should the
nurse take? (Select all that apply.)
A. Notify the healthcare provider of the client’s change in mental status.
B. Include q2 hour’s reorientation in the client’s plan of care.
Answer: A. Notify the healthcare provider of the client’s change in mental status.
B. Include q2 hour’s reorientation in the client’s plan of care.
119. An older male comes to the clinic with a family member. When the nurse attempts to
take the client’s health history, he does not respond to questions in a clear manner. What
action should the nurse implement first?
Answer: Assess the surroundings for noise and distractions.
120. The nurse caring for a client with acute renal fluid (ARF) has noted that the client has
voided 800 ml of urine in 4 hours. Based on this assessment, what should the nurse
anticipate, that client will need?
Answer: Large amounts of fluid and electrolyte replacement.
121. Which intervention should the nurse include in the plan of care for a child with tetanus?

Answer: Minimize the amount of stimuli in the room
122. Suicide precautions are initiated for a child admitted to the mental health unit following
an intentional narcotic overdose. After a visitor leaves, the nurse finds a package of cigarettes
in the client’s room. Which intervention is most important for the nurse to implement?
Answer: Remove cigarettes for the client’s room
123. A family member of a frail elderly adult asks the nurse about eligibility requirements for
hospice care. What information should the nurse provide? (Select all that apply.)
A. A client must be willing to accept palliative care, not curative care.
B. The healthcare provider must project that the client has 6 months or less to live.
Answer: A. A client must be willing to accept palliative care, not curative care.
B. The healthcare provider must project that the client has 6 months or less to live.
124. A client with atrial fibrillation receives a new prescription for dabigatran. What
instruction should the nurse include in this client’s teaching plan?
Answer: Avoid use of nonsteroidal ant-inflammatory drugs (NSAID).
125. A nurse with 10 years’ experience working in the emergency room is reassigned to the
perinatal unit to work an 8 hour shift. Which client is best to assign to this nurse?
Answer: A mother with an infected episiotomy
126. An infant who is admitted for surgical repair of a ventricular septal defect (VSD) is
irritable and diaphoretic with jugular vein distention. Which prescription should the nurse
administer first?
Answer: Digoxin.
127. The nursing staff on a medical unit includes a registered nurse (RN), practical nurse
(PN), and an unlicensed assistive personnel (UAP). Which task should the charge nurse
assign to the RN?
Answer: Supervise a newly hired graduate nurse during an admission assessment.

128. While teaching a young male adult to use an inhaler for his newly diagnosed asthma, the
client stares into the distance and appears to be concentrating on something other than the
lesson the nurse is presenting. What action should the nurse take?
Answer: Ask the client what he is thinking about at his time.
129. After several hours of non-productive coughing, a client presents to the emergency room
complaining of chest tightness and shortness of breath. History includes end stage chronic
obstructive pulmonary disease (COPD) and diabetes mellitus. While completing the
pulmonary assessment, the nurse hears wheezing and poor air movement bilaterally. Which
actions should the nurse implement? (Select all that apply.)
A. Administer PRN nebulizer treatment.
B. Obtain 12 lead electrocardiogram.
C. Monitor continuous oxygen saturation.
Answer: A. Administer PRN nebulizer treatment.
B. Obtain 12 lead electrocardiogram.
C. Monitor continuous oxygen saturation.
130. The nurse caring for a 3-month-old boy one day after a pylorotomy notices that the
infant is restless, is exhibiting facial grimaces, and is drawing his knees to his chest. What
action should the nurse take?
Answer: Administer a prescribed analgesia for pain.
131. A 4-year-old with acute lymphocytic leukemia (ALL) is receiving a chemotherapy (CT)
protocol that includes methotrexate (Mexate, Trexal, MIX), an antimetabolite. Which
information should the nurse provide the parents about caring for their child?
Answer: Use sunblock or protective clothing when outdoors.
132. Two days after admission a male client remembers that he is allergic to eggs, and
informs the nurse of the allergy. Which actions should the nurse implement? (Select all that
apply)
A. Notify the food services department of the allergy.
B. Enter the allergy information in the client’s record.
C. Add egg allergy to the client’s allergy arm band.
Answer: A. Notify the food services department of the allergy.

B. Enter the allergy information in the client’s record.
C. Add egg allergy to the client’s allergy arm band.
133. The rapid response team’s detects return of spontaneous circulation (ROSC) after 2 min
of continuous chest compressions. The client has a weak, fast pulse and no respiratory effort,
so the healthcare provider performs a successful oral, intubation. What action should the
nurse implement?
Answer: Perform bilateral chest auscultation.
134. After administering an antipyretic medication. Which intervention should the nurse
implement?
Answer: Encouraging liberal fluid intake
135. A client with hyperthyroidism is being treated with radioactive iodine (I-131).
Which explanation should be included in preparing this client for this treatment?
Answer: Describe radioactive iodine as a tasteless, colorless medication administered by the
healthcare provider
136. After a colon resection for colon cancer, a male client is moaning while being transferred
to the Postanesthesia Care Unit (PACU). Which intervention should the nurse implement
first?
Answer: Determine client’s pulse, blood pressure, and respirations
137. The nurse is caring for a group of clients with the help of a licensed practical nurse
(LPN) and an experienced unlicensed assistive personnel (UAP). Which procedures can the
nurse delegate to the UAP? (Select all that apply)
A. Take postoperative vital signs for a client who has an epidual following knee arthroplasty
B. Collect a sputum specimen for a client with a fever of unknown origin
C. Ambulate a client who had a femoral-popliteal bypass graft yesterday
Answer: A. Take postoperative vital signs for a client who has an epidual following knee
arthroplasty
B. Collect a sputum specimen for a client with a fever of unknown origin
C. Ambulate a client who had a femoral-popliteal bypass graft yesterday

138. A male client with cirrhosis has ascites and reports feeling short of breath. The client is
in semi Fowler position with his arms at his side. What action should the nurse implement?
Answer: Raise the head of the bed to a Fowler’s position and support his arms with a pillow
139. A client with a history of chronic pain requests a nonopioid analgesic. The client is alert
but has difficulty describing the exact nature and location of the pain to the nurse. Which
action should the nurse implement next?
Answer: Administer the analgesic as requested
Rationale:
Chronic pain may be difficult to describe but should be treated with analgesics as indicated.
140. A client with a chronic health problem has difficulty ambulating short distance due to
generalized weakness, but is able to bear weight on both legs. To assist with ambulation and
provide the greatest stability, what assistive device is best for this client?
A. Crutches with 2 point gait.
B. Crutches with 3 point gait.
C. Crutches with 4 point gait.
D. A quad cane
Answer: C. Crutches with 4 point gait.
141. The nurse uses the parkland formula (4ml x kg x total body surface area = 24 hours fluid
replacement) to calculate the 24-hours IV fluid replacement for a client with 40% burns who
weighs 76kg. How many ml should the client receive? (Enter numeric value only.)
Answer: 12160
Rationale:
4ml x 67kg x 40 (bsa) = 12,160 ml
142. A client with leukemia undergoes a bone marrow biopsy. The client’s laboratory values
indicate the client has thrombocytopenia. Based on this data, which nursing assessment is
most important following the procedure?
A. Observe aspiration site.
B. Assess body temperature
C. Monitor skin elasticity
D. Measure urinary output

Answer: A. Observe aspiration site.
143. An 18-year-old female client is seen at the health department for treatment of
condylomata acuminate (perineal warts) caused by the human papillomavirus (HPV). Which
intervention should the nurse implement?
Answer: Reinforce the importance of annual papanicolaou (Pap) smears.
144. A client admitted to the psychiatric unit diagnosed with major depression wants to sleep
during the day, refuses to take a bath, and refuses to eat. Which nursing intervention should
the nurse implement first?
Answer: Establish a structured routine for the client to follow.
145. A client with history of bilateral adrenalectomy is admitted with a week, irregular pulse,
and hypotension. Which assessment finding warrants immediate intervention by the nurse?
Answer: Ventricular arrhythmias.
Rationale:
Adrenal crisis, a potential complication of bilateral adrenalectomy, results in the loss of
mineralocorticoids and sodium excretions that is characterized by hyponatremia,
hyperkalemia, dehydration, and hypotension. Ventricular arrhythmias are life threatening and
required immediate intervention to correct critical potassium levels.
146. The mother of a 7-month-old brings the infant to the clinic because the skin in the diaper
area is excoriated and red, but there are no blisters or bleeding. The mother reports no
evidence of watery stools. Which nursing intervention should the nurse implement?
Answer: Instruct the mother to change the child’s diaper more often.
147. A resident of a long-term care facility, who has moderate dementia, is having difficulty
eating in the dining room. The client becomes frustrated when dropping utensils on the floor
and then refuses to eat. What action should the nurse implement?
Answer: Encourage the client to eat finger foods.
148. A client is receiving mesalamine 800 mg PO TID. Which assessment is most important
for the nurse to perform to assess the effectiveness of the medication?
Answer: Bowel patterns

Rationale:
The client should be assessed for a change in bowel patterns to evaluate the effectiveness of
this medication because Mesalamine is used to treat ulcerative colitis (a condition which
causes swelling and sores in the lining of the colon [large intestine] and rectum) and also to
maintain improvement of ulcerative colitis symptoms. Mesalamine is in a class of
medications called anti-inflammatory agents. It works by stopping the body from producing a
certain substance that may cause inflammation.
149. While in the medical records department, the nurse observes several old medical records
with names visible in waste container. What action should the nurse implement?
Answer: Contact the medical records department supervisor.
150. A 16-year-old adolescent with meningococcal meningitis is receiving a continuous IV
infusion of penicillin G, which is prescribed as 20 million units in a total volume of 2 liters of
normal saline every 24 hr. The pharmacy delivers 10 million units/ liters of normal saline.
How many ml/hr should the nurse program the infusion pump? (Enter numeric value only. If
rounding is required, round to the nearest whole number.)
Answer: 83
Rationale:
1000 ml-----12hr.
Xml ---------1hr.
1000/12 = 83.33 = 83.
151. While visiting a female client who has heart failure (HF) and osteoarthritis, the home
health nurse determines that the client is having more difficulty getting in and out of the bed
than she did previously. Which action should the nurse implement first?
Answer: Submit a referral for an evaluation by a physical therapist.
152. A client has an intravenous fluid infusing in the right forearm. To determine the client’s
distal pulse rate most accurately, which action should the nurse implement?
Answer: Palpate at the radial pulse site with the pads of two or three fingers.

153. A child is admitted to the pediatric unit diagnosed with sickle cell crisis. When the nurse
walks into the room, the unlicensed assistive personnel (UAP) is encouraging the child to
stay in bed in the supine position. Which action should the nurse implement?
Answer: Reposition the client with the head of the bed elevated.
154. A preschool-aged boy is admitted to the pediatric unit following successful resuscitation
from a near-drowning incident. While providing care to child, the nurse begins talking with
his preadolescent brother who rescued the child from the swimming pool and initiated
resuscitation. The nurse notices the older boy becomes withdrawn when asked about what
happened. What action should the nurse take?
Answer: Ask the older brother how he felt during the incident.
155. After six days on a mechanical ventilator, a male client is extubated and place on 40%
oxygen via face mask. He is awake and cooperative, but complaining of a severe sore throat.
While sipping water to swallow a medication, the client begins coughing, as if strangled.
What intervention is most important for the nurse to implement?
Answer: Hold oral intake until swallow evaluation is done.
156. The nurse is interacting with a female client who is diagnosed with postpartum
depression. Which finding should the nurse document as an objective signs of depression?
(Select all that apply)
A. Interacts with a flat affect.
B. Avoids eye contact.
C. Has a disheveled appearance.
Answer: A. Interacts with a flat affect.
B. Avoids eye contact.
C. Has a disheveled appearance.
157. A client in the postanaesthetic care unit (PACU) has an eight (8) on the Aldrete
postanaesthetic scoring system. What intervention should nurse implement?
Answer: Transfer the client to the surgical floor.
158. In caring for the body of a client who just died, which tasks can be delegate to the
unlicensed assistive personnel (UAP)? (Select all that apply.)

A. Place personal religious artifacts on the body.
B. Attach identifying name tags to the body.
C. Follow cultural beliefs in preparing the body.
Answer: A. Place personal religious artifacts on the body.
B. Attach identifying name tags to the body.
C. Follow cultural beliefs in preparing the body.
159. An adult male reports the last time he received penicillin he developed a severe
maculopapular rash all over his chest. What information should the nurse provide the client
about future antibiotic prescriptions?
Answer: Be alert for possible cross-sensitivity to cephalosporin agents.
160. A client with a prescription for “do not resuscitate” (DNR) begins to manifest signs of
impending death. After notifying the family of the client’s status, what priority action should
the nurse implement?
Answer: The client’s need for pain medication should be determined.
161. A client with cirrhosis of the liver is admitted with complications related to end stage
liver disease. Which intervention should the nurse implement? (Select all that apply.)
A. Monitor abdominal girth.
B. Increase oral fluid intake to 1500 ml daily.
C. Report serum albumin and globulin levels.
D. Provide diet low in phosphorous.
E. Note signs of swelling and edema.
Answer: A. Monitor abdominal girth.
C. Report serum albumin and globulin levels.
E. Note signs of swelling and edema.
Rationale:
Monitoring for increasing abdominal girth and generalized tissue edema and swelling are
focused assessments that provide data about the progression of disease related complications.
In advanced cirrhosis, liver function failure results in low serum albumin and serum protein
levels, which caused third spacing that results in generalized fluid retention and ascites. Other
options are not indicated in end stage liver disease.

162. During discharge teaching, the nurse discusses the parameters for weight monitoring
with a client who was recently diagnosed with heart failure (HF). Which information is most
important for the client to acknowledge?
Answer: Report weight gain of 2 pounds (0.9kg) in 24 hours
163. Which problem, noted in the client’s history, is important for the nurse to be aware of
prior to administration of a newly prescribed selective serotonin reuptake inhibitor (SSRI)?
Answer: Aural migraine headaches.
164. When implementing a disaster intervention plan, which intervention should the nurse
implement first?
A. Initiate the discharge of stable clients from hospital units
B. Identify a command center where activities are coordinated
C. Assess community safety needs impacted by the disaster
D. Instruct all essential off-duty personnel to report to the facility
Answer: B. Identify a command center where activities are coordinated
165. The nurse is evaluating a client’s symptoms, and formulates the nursing diagnosis, “high
risk for injury due to possible urinary tract infection.” Which symptoms indicate the need for
this diagnosis?
Answer: Fever and dysuria.
166. A client is admitted with metastatic carcinoma of the liver, ascites, and bilateral 4+
pitting edema of both lower extremities. When the client complains that the antiembolic
stocking are too constricting, which intervention should the nurse implement?
Answer: Maintain both lower extremities elevated on pillows.
167. A client with muscular dystrophy is concerned about becoming totally dependent and is
reluctant to call the nurse to assist with activities of daily living (ADLs). To achieve
maximum mobility and independence, which intervention is most important for the nurse to
include in the client’s plan of care?
Answer: Teach family proper range of motion exercises.

168. The nurse is teaching a postmenopausal client about osteoporosis prevention. The client
reports that she smokes 2 packs of cigarettes a day and takes 750 mg calcium supplements
daily. What information should the nurse include when teaching this client about osteoporosis
prevention?
Answer: Postmenopausal women need an intake of at least 1,500 mg of calcium daily.
169. When evaluating a client’s rectal bleeding, which findings should the nurse document?
Answer: Color characteristics of each stool.
170. The nurse is auscultating a client’s lung sounds. Which description should the nurse use
to document this sound?
A. High pitched or fine crackles.
B. Rhonchi
C. High pitched wheeze
D. Stridor
Answer: A. High pitched or fine crackles.
171. An adult male is admitted to the emergency department after falling from a ladder. While
waiting to have a computed tomography (CT) scan, he requests something for a severe
headache. When the nurse offers him a prescribed does of acetaminophen, he asks for
something stronger. Which intervention should the nurse implement?
Answer: Explain the reason for using only non-narcotics.
172. The nurse is managing the care of a client with Cushing’s syndrome. Which
interventions should the nurse delegate to the unlicensed assistive personnel (UAP)? (Select
all that apply)
A. Weigh the client and report any weight gain.
B. Report any client complaint of pain or discomfort.
C. Note and report the client’s food and liquid intake during meals and snacks.
Answer: A. Weigh the client and report any weight gain.
B. Report any client complaint of pain or discomfort.
C. Note and report the client’s food and liquid intake during meals and snacks.

173. Ten years after a female client was diagnosed with multiple sclerosis (MS), she is
admitted to a community palliative care unit. Which intervention is most important for the
nurse to include in the client’s plan of care?
Answer: Medicate as needed for pain and anxiety.
174. An increased number of elderly persons are electing to undergo a new surgical procedure
which cures glaucoma. What effect is the nurse likely to note as a result of this increases in
glaucoma surgeries?
Answer: Decrease prevalence of glaucoma in the population.
175. The nurse is caring for a client who is entering the second stage of labor. Which action
should the nurse implement first?
Answer: Convey to the client that birth is imminent.
176. To evaluate the effectiveness of male client’s new prescription for ezetimibe, which
action should the clinic nurse implement?
Answer: Remind the client to keep his appointments to have his cholesterol level checked.
177. Diagnostic studies indicate that the elderly client has decreased bone density. In
providing client teaching, which area of instruction is most important for the nurse to
include?
Answer: Fall prevention measures.
178. A young adult client is admitted to the emergency room following a motor vehicle
collision. The client’s head hit the dashboard. Admission assessment include: Blood pressure
85/45 mm Hg, temperature 98.6 F, pulse 124 beat/minute and respirations 22 breath/minute.
Based on these data, the nurse formulates the first portion of nursing diagnosis as ” Risk of
injury” What term best expresses the “related to” portion of nursing diagnosis?
A. Infection
B. Increase intracranial pressure
C. Shock
D. Head Injury.
Answer: C. Shock

179. An older male client with history of diabetes mellitus, chronic gout, and osteoarthritis
comes to the clinic with a bag of medication bottles. Which intervention should the nurse
implement first?
Answer: Identify pills in the bag.
180. A male client who was diagnosed with viral hepatitis A 4 weeks ago returns to the clinic
complaining of weakness and fatigue. Which finding is most important for the nurse to report
to the healthcare provider?
Answer: New onset of purple skin lesions.
181. In assessing a client twelve hours following transurethral resection of the prostate
(TURP), the nurse observes that the urinary drainage tubing contains a large amount of clear
pale pink urine and the continuous bladder irrigation is infusing slowly. What action should
the nurse implement?
Answer: Ensure that no dependent loops are present in the tubing.
182. The healthcare provider prescribes the antibiotic Cefdinir (cephalosporin) 300mg PO
every 12 h for a client with postoperative wound infections. Which feeds should the nurse
encourage this client to eat?
A. Yogurt and/or buttermilk.
B. Avocados and cheese
C. Green leafy vegetables
D. Fresh fruits
Answer: A. Yogurt and/or buttermilk.
183. The charge nurse is making assignment on a psychiatric unit for a practical nurse (PN)
and newly license register nurse (RN). Which client should be assigned to the RN?
A. An adult female who has been depress for the past several month and denies suicidal
ideations.
B. A middle-age male who is in depressive phase on bipolar disease and is receiving Lithium.
C. A young male with schizophrenia who said voices is telling him to kill his psychiatric.
D. An elderly male who tell the staff and other client that he is superman and he can fly.
Answer: C. A young male with schizophrenia who said voices is telling him to kill his
psychiatric.

Rationale:
The RN should deal with the client with command hallucinations and these can be very
dangerous if the client’s acts on the commands, especially if the command is a homicidal in
nature. Other client present low safety risk.
184. A client at 30 week gestation is admitted due to preterm labor. A prescription of
terbutaline sulfate 8.35 mg is gives subcutaneously. Based on which finding should the nurse
withhold the next dose of this drug?
Answer: Maternal pulse rate of 162 beats per min
185. In assessing an older female client with complication associated with chronic obstructive
pulmonary disease (COPD), the nurse notices a change in the client’s appearance. Her face
appears tense and she begs the nurse not to leave her alone. Her pulse rate is 100, and
respirations are 26 per min. What is the primary nursing diagnosis?
Answer: Anxiety related to fear of suffocation.
186. A client with a cervical spinal cord injury (SCI) has Crutchfield tongs and skeletal
traction applied as a method of closed reduction. Which intervention is most important for the
nurse to include in the client’s a plan of care?
Answer: Provide daily care of tong insertion sites using saline and antibiotic ointment
187. A client arrives on the surgical floor after major abdominal surgery. What intervention
should the nurse perform first?
Answer: Determine the client’s vital sign.
188. A client is admitted to the emergency department with a respiratory rate of 34 breaths
per minute and high pitched wheezing on inspiration and expiration, the medical diagnosis is
severe exacerbation of asthma. Which assessment finding, obtained 10 min after the
admission assessment, should the nurse report immediately to the emergency department
healthcare provider?
Answer: No wheezing upon auscultation of the chest.

189. The nurse is planning a class for a group of clients with diabetes mellitus about blood
glucose monitoring. In teaching the class as a whole, the nurse should emphasize the need to
check glucose levels in which situation?
Answer: During acute illness
190. A 350-bed acute care hospital declares an internal disaster because the emergency
generators malfunctioned during a city-wide power failure. The UAPs working on a general
medical unit ask the charge nurse what they should do first. What instruction should the
charge nurse provide to these UAPs?
Answer: Tell all their assigned clients to stay in their rooms.
191. The nurse is auscultating is auscultating a client’s heart sounds. Which description
should the nurse use to document this sound? (Please listen to the audio file to select the
option that applies.)
A. Murmur
B. s1 s2
C. pericardial friction rub
D. s1 s2 s3
Answer: A. Murmur
192. The healthcare provider changes a client’s medication prescription from IV to PO
administration and double the dose. The nurse notes in the drug guide that the prescribed
medication, when given orally, has a high first-pass effect and reduce bioavailability. What
action should the nurse implement?
Answer: Administer the medication via the oral route as prescribed
193. A client refuses to ambulate, reporting abdominal discomfort and bloating caused by
“too much gas buildup” the client’s abdomen is distended. Which prescribed PRN medication
should the nurse administer?
Answer: Simethicone (Mylicon)
194. The public nurse health received funding to initiate primary prevention program in the
community. Which program the best fits the nurse’s proposal?
A. Case management and screening for clients with HIV.

B. Regional relocation center for earthquake victims
C. Vitamin supplements for high-risk pregnant women.
D. Lead screening for children in low-income housing.
Answer: C. Vitamin supplements for high-risk pregnant women.
Rationale:
Primary prevention activities focus on health promotions and disease preventions, so vitamin
for high-risk pregnant women provide adequate vitamin and mineral for fetal developmental.
195. When assessing and adult male who presents as the community health clinic with a
history of hypertension, the nurse note that he has 2+ pitting edema in both ankles. He also
has a history of gastroesophageal reflex disease (GERD) and depression. Which intervention
is the most important for the nurse to implement?
A. Arrange to transport the client to the hospital
B. Instruct the client to keep a food journal, including portions size.
C. Review the client’s use of over the counter (OTC) medications.
D. Reinforce the importance of keeping the feet elevated.
Answer: C. Review the client’s use of over the counter (OTC) medications.
Rationale:
Sodium is used in several types of OTC medications. Including antacids, which the client
may be using to treat his GERD. Further evaluation is need it to determine the need for
hospitalization (A) A food journal (B) may help over, but dietary modifications are needed
now since edema is present. (C) May relieve dependent edema, but not treat the underlying
etiology.
196. An older client is admitted to the intensive care unit with severe abdominal pain,
abdominal distention, and absent bowel sound. The client has a history of smoking 2 packs of
cigarettes daily for 50 years and is currently restless and confused. Vital signs are:
temperature 96`F, heart rate 122 beats/minute, respiratory rate 36 breaths/minute, mean
arterial pressure(MAP) 64 mmHg and central venous pressure (CVP) 7 mmHg. Serum
laboratory findings include: hemoglobin 6.5 grams/dl, platelets 6o, 000, and white blood cell
count (WBC) 3,000/mm3. Based on these findings this client is at greatest risk for which
pathophysiological condition?
A. Multiple organ dysfunction syndrome (MODS)
B. Disseminated intravascular coagulation (DIC)

C. Chronic obstructive disease.
D. Acquired immunodeficiency syndrome (AIDS)
Answer: A. Multiple organ dysfunction syndrome (MODS)
Rationale:
MODS are a progressive dysfunction of two or more major organs that requires medical
intervention to maintain homeostasis. This client has evidence of several organ systems that
require intervention, such as blood pressure, hemoglobin, WBC, and respiratory rate. DIC
may develop as a result of MODS. The other options are not correct.
197. A man expresses concern to the nurse about the care his mother is receiving while
hospitalized. He believes that her care is not based on any ethical standards and ask what type
of care he should expect from a public hospital. What action should the nurse take?
Answer: Provide the man and his mother with a copy of the Patient’s Bill of Rights
198. A client experiencing withdrawal from the benzodiazepines alprazolam (Xanax) is
demonstrating severe agitation and tremors. What is the best initial nursing action?
A. Administer naloxone (Narcan) per PNR protocol
B. Initiate seizure precautions
C. Obtain a serum drug screen
D. Instruct the family about withdrawal symptoms.
Answer: B. Initiate seizure precautions
Rationale:
Withdrawal of CNS depressants, such as Xanax, results in rebound over-excitation of the
CNS. Since the client exhibiting tremors, the nurse should anticipate seizure activity and
protect the client.
199. The nurse is caring for a client who is taking a macrolide to treat a bacterial infection.
Which finding should the nurse report to the healthcare provider before administering the
next dose?
A. Jaundice
B. Nausea
C. Fever
D. Fatigue
Answer: A. Jaundice

200. A client with Alzheimer’s disease (AD) is receiving trazodone (Desyrel), a recently
prescribed atypical antidepressant. The caregiver tells the home health nurse that the client’s
mood and sleep patterns are improved, but there is no change in cognitive ability. How
should the nurse respond to this information?
A. Explain that it may take several weeks for the medication to be effective
B. Confirm the desired effect of the medication has been achieved.
C. Notify the health care provider than a change may be needed.
D. Evaluate when and how the medication is being administered to the client.
Answer: B. Confirm the desired effect of the medication has been achieved.
Rationale:
Trazodone o Desyrel, an atypical antidepressant, is prescribed for client with AD to improve
mood and sleep.
201. A client with diabetic peripheral neuropathy has been taking pregabalin (Lyrica) for 4
days. Which finding indicates to the nurse that the medication is effective?
A. Reduced level of pain
B. Full volume of pedal pulses
C. Granulating tissue in foot ulcer
D. Improved visual acuity
Answer: A. Reduced level of pain
202. A group of nurse-managers is asked to engage in a needs assessment for a piece of
equipment that will be expensed to the organization’s budget. Which question is most
important to consider when analyzing the cost-benefit for this piece of equipment?
A. How many departments can use this equipment?
B. Will the equipment require annual repair?
C. Is the cost of the equipment reasonable?
D. Can the equipment be updated each year?
Answer: A. How many departments can use this equipment?
203. While receiving a male postoperative client’s staples de nurse observe that the client’s
eyes are closed and his face and hands are clenched. The client states, “I just hate having

staples removed”. After acknowledgement the client’s anxiety, what action should the nurse
implement?
A. Encourage the client to continue verbalize his anxiety
B. Attempt to distract the client with general conversation
C. Explain the procedure in detail while removing the staples
D. Reassure the client that this is a simple nursing procedure.
Answer: B. Attempt to distract the client with general conversation
Rationale:
Distract is an effective strategy when a client experience anxiety during an uncomfortable
procedure. (A & D) increase the client’s anxiety.
204. A male client is admitted for the removal of an internal fixation that was inserted for the
fracture ankle. During the admission history, he tells the nurse he recently received
vancomycin (vancomycin) for a methicillin-resistant Staphylococcus aureus (MRSA) wound
infection. Which action should the nurse take? (Select all that apply.)
A. Collect multiple site screening culture for MRSA
B. Call healthcare provider for a prescription for linezolid (Zyrovix)
C. Place the client on contact transmission precautions
D. Obtain sputum specimen for culture and sensitivity
E. Continue to monitor for client sign of infection.
Answer: A. Collect multiple site screening culture for MRSA
C. Place the client on contact transmission precautions
E. Continue to monitor for client sign of infection.
Rationale:
Until multi-site screening cultures come back negative (A), the client should be maintained
on contact isolation(C) to minimize the risk for nosocomial infection. Linezolid (Zyvox), a
broad spectrum anti-infectant, is not indicated, unless the client has an active skin structure
infection cause by MRSA or multidrug- resistant strains (MDRSP) of Staphylococcus aureus.
A sputum culture is not indicated9D) based on the client’s history is a wound infection.
205. A vacuum-assistive closure (VAC) device is being use to provide wound care for a client
who has stage III pressure ulcer on a below-the- knee (BKA) residual limb. Which
intervention should the nurse implement to ensure maximum effectiveness of the device?
Answer: Ensure the transparent dressing has no tears that might create vacuum leaks

206. The nurse is developing the plan of care for a client with pneumonia and includes the
nursing diagnosis of “Ineffective airway clearance related to thick pulmonary secretions.”
Which intervention is most important for the nurse to include in the client’s plan of care?
Answer: Increase fluid intake to 3,000 ml/daily
207. The nurse plans to collect a 24- hour urine specimen for a creatinine clearance test.
Which instruction should the nurse provide to the adult male client?
A. Clearance around the meatus, discard first portion of voiding, and collect the rest in a
sterile bottle
B. Urinate at specific time, discard the urine, and collect all subsequent urine during the next
24 hours.
C. For the next 24 hours, notify the nurse when the bladder is full, and the nurse will collect
catheterized specimens.
D. Urinate immediately into a urinal, and the lab will collect specimen every 6 hours, for the
next 24 hours.
Answer: B. Urinate at specific time, discard the urine, and collect all subsequent urine during
the next 24 hours.
Rationale:
Urinate at specific time, discard the urine, and collect all subsequent urine during the next 24
hours is the correct procedure for collecting 24-hour urine specimen. Discarding even one
voided specimen invalidate the test.
208. The nurse is preparing to administer a histamine 2-receptor antagonist to a client with
peptic ulcer disease. What is the primary purpose of this drug classification?
Answer: Decreases the amount of HCL secretion by the parietal cells in the stomach
209. The healthcare provider prescribes acarbose (Precose), an alpha-glucosidase inhibitor,
for a client with Type 2 diabetes mellitus. Which information provides the best indicator of
the drug’s effectiveness?
Answer: Hemoglobin A1C (HbA1C) reading less than 7%
210. The nurse assesses a client with new onset diarrhoea. It is most important for the nurse
to question the client about recent use of which type of medication?

A. Antibiotics
B. Anticoagulants
C. Antihypertensive
D. Anticholinergics
Answer: A. Antibiotics
211. A neonate with a congenital heart defect (CHD) is demonstrating symptoms of heart
failure (HF). Which interventions should the nurse include in the infant’s plan of care?
A. Give O2 at 6 L/nasal cannula for 3 repeated oximetry screens below 90%
B. Administer diuretics via secondary infusion in the morning only
C. Evaluate heart rate for effectiveness of cardio tonic medications
D. Use high energy formula 30 calories/ounce at Q3 hours feeding via soft nipples
E. Ensure Interrupted and frequent rest periods between procedures.
Answer: A. Give O2 at 6 L/nasal cannula for 3 repeated oximetry screens below 90%
C. Evaluate heart rate for effectiveness of cardio tonic medications
D. Use high energy formula 30 calories/ounce at Q3 hours feeding via soft nipples
E. Ensure Interrupted and frequent rest periods between procedures.
Rationale:
Pulse oximetry screening supports prescribed level of O2. HR provides an evaluative criterion
for cardiac medications, which reduce heart rate, increase strength contractions (inotropic
effects) and consequently affect systemic circulation and tissue oxygenation. Breast milk or
basic formula provide 20 calories/ounce, so frequent feedings with high energy formula. D
minimize fatigue is necessary.
212. The nurse is caring for a 4-year-old male child who becomes unresponsive as his heart
rate decreases to 40 beats/minute. His blood pressure is 88/70 mmHg, and his oxygen
saturation is 70% while receiving 100% oxygen by non-rebreather face mask. In what
sequence, from first to last, should the nurse implement these actions? (Place the first action
on top and last action on the bottom.)
1. Start chest compressions with assisted manual ventilations
2. Administer epinephrine 0.01 mg/kg intraosseous (IO)
3. Apply pads and prepare for transthoracic pacing
4. Review the possible underlying causes for bradycardia
Answer:

1. Start chest compressions with assisted manual ventilations
3. Apply pads and prepare for transthoracic pacing
2. Administer epinephrine 0.01 mg/kg intraosseous (IO)
4. Review the possible underlying causes for bradycardia
213. An elderly male client is admitted to the mental health unit with a sudden onset of global
disorientation and is continuously conversing with his mother, who died 50 years ago. The
nurse reviews the multiple prescriptions he is currently taking and assesses his urine
specimen, which is cloudy, dark yellow, and has foul Odor. These findings suggest that his
client is experiencing which condition?
A. Delirium
B. Depression
C. Dementia
D. Psychotic episode
Answer: A. Delirium
214. Following an esophagogastroduodenoscopy (EGD) a male client is drowsy and difficult
to arouse, and his respiration are slow and shallow. Which action should the nurse
implement? Select all that apply.
A. Prepare medication reversal agent
B. Check oxygen saturation level
C. Apply oxygen via nasal cannula
D. Initiate bag- valve mask ventilation.
E. Begin cardiopulmonary resuscitation
Answer: A. Prepare medication reversal agent
B. Check oxygen saturation level
C. Apply oxygen via nasal cannula
Rationale:
Sedation, given during the procedure may need to be reverse if the client does not easily
wake up. Oxygen saturation level should be asses, and oxygen applied to support respiratory
effort and oxygenation. The client is still breathing so the bag- valve mask ventilation and
CPR are not necessary.

215. The nurse is planning preoperative teaching plan of a 12-years old child who is
scheduled for surgery. To help reduce the child anxiety, which action is the best for the nurse
to implement?
A. Give the child syringes or hospital mask to play it at home prior to hospitalization.
B. Include the child in pay therapy with children who are hospitalized for similar surgery.
C. Provide a family tour of the preoperative unit one week before the surgery is scheduled.
D. Provide doll an equipment to re-enact feeling associated with painful procedures.
Answer: C. Provide a family tour of the preoperative unit one week before the surgery is
scheduled.
Rationale:
School age children gain satisfaction from exploring and manipulating their environment,
thinking about objectives, situations and events, and making judgments based on what they
reason. A tour of the unit allows the child to see the hospital environment and reinforce
explanation and conceptual thinking.
216. Which intervention should the nurse implement during the administration of vesicant
chemotherapeutic agent via an IV site in the client’s arm?
Answer: Assess IV site frequently for signs of extravasation
217. When development a teaching plan for a client newly diagnosed type 1 diabetes, the
nurse should explain that an increase thirst is an early sing of diabetes ketoacidosis (DKA),
which action should the nurse instruct the client to implement if this sign of DKA occur?
A. Resume normal physical activity
B. Drink electrolyte fluid replacement
C. Give a dose of regular insulin per sliding scale
D. Measure urinary output over 24 hours.
Answer: C. Give a dose of regular insulin per sliding scale
Rationale:
As hyperglycaemia persist, ketone body become a fuel source, and the client manifest early
signs of DKA that include excessive thirst, frequent urination, headache, nausea and
vomiting. Which result in dehydration and loss of electrolyte. The client should determine
fingerstick glucose level and self-administer a dose of regular insulin per sliding scale.

218. The nurse is teaching a group of clients with rheumatoid arthritis about the need to
modify daily activities. Which goal should the nurse emphasize?
A. Protect joint function
B. Improve circulation
C. Control tremors
D. Increase weight bearing
Answer: A. Protect joint function
219. An adult client experiences a gasoline tank fire when riding a motorcycle and is admitted
to the emergency department (ED) with full thickness burns to all surfaces of both lower
extremities. What percentage of body surface area should the nurse document in the
electronic medical record (EMR)?
A. 9 %
B. 18 %
C. 36 %
D. 45 %
Answer: C. 36 %
Rational: according to the rule of nines, the anterior and posterior surfaces of one lower
extremity is designated as 18 %of total body surface area (TBSA), so both extremities equals
36% TBSA, other options are incorrect.
220. A client with hyperthyroidism is receiving propranolol (Inderal). Which finding
indicates that the medication is having the desired effect?
A. Decrease in serum T4 levels
B. Increase in blood pressure
C. Decrease in pulse rate
D. Goiter no longer palpable
Answer: C. Decrease in pulse rate
221. An older male client with type 2 diabetes mellitus reports that has experiences legs pain
when walking short distances, and that the pain is relieved by rest. Which client behavior
indicates an understanding of healthcare teaching to promote more effective arterial
circulation?
A. Consistently applies TED hose before getting dressed in the morning.

B. Frequently elevated legs thorough the day.
C. Inspect the leg frequently for any irritation or skin breakdown
D. Completely stop cigarette/ cigar smoking.
Answer: D. Completely stop cigarette/ cigar smoking.
Rationale:
Stopping cigarette smoking helps to decrease vasoconstriction and improve arterial
circulation to the extremity.
222. A community health nurse is concerned about the spread of communicable diseases
among migrant farm workers in a rural community. What action should the nurse take to
promote the success of a healthcare program designed to address this problem?
Answer: Establish trust with community leaders and respect cultural and family values
223. The nurse performs a prescribed neurological check at the beginning of the shift on a
client who was admitted to the hospital with a subarachnoid brain attack (stroke). The client’s
Glasgow Coma Scale (GCS) score is 9. What information is most important for the nurse to
determine?
A. The client’s previous GCS score
B. When the client’s stroke symptoms started
C. If the client is oriented to time
D. The client’s blood pressure and respiration rate
Answer: A. The client’s previous GCS score
Rationale:
The normal GCS is 15, and it is most important for the nurse to determine if it abnormal
score a sign of improvement or a deterioration in the client’s condition
224. The charge nurse in a critical care unit is reviewing clients’ conditions to determine who
is stable enough to be transferred. Which client status report indicates readiness for transfer
from the critical care unit to a medical unit?
Answer: Chronic liver failure with a hemoglobin of 10.1 and slight bilirubin elevation
225. Based on principles of asepsis, the nurse should consider which circumstance to be
sterile?
A. One inch- border around the edge of the sterile field set up in the operating room

B. A wrapped unopened, sterile 4 x 4 gauze placed on a damp table top.
C. An open sterile Foley catheter kit set up on a table at the nurse waist level
D. Sterile syringe is placed on sterile area as the nurse riches over the sterile field.
Answer: C. An open sterile Foley catheter kit set up on a table at the nurse waist level
Rationale:
A sterile package at or above the waist level is considered sterile. The edge of sterile field is
contaminated which include a 1-inch border (A). A sterile objects become contaminated by
capillary action when sterile objects become in contact with a wet contaminated surface.
226. An unlicensed assistive personnel (UAP) reports that a client’s right hand and fingers
spasms when taking the blood pressure using the same arm. After confirming the presence of
spams what action should the nurse take?
A. Ask the UAP to take the blood pressure in the other arm
B. Tell the UAP to use a different sphygmomanometer.
C. Review the client’s serum calcium level
D. Administer PRN antianxiety medication.
Answer: C. Review the client’s serum calcium level
Rationale:
Trousseau’s sign is indicated by spasms in the distal portion of an extremity that is being used
to measure blood pressure and is caused by hypocalcemia (normal level 9.0-10.5 mg/dl, so C
should be implemented.
227. A 56-years-old man shares with the nurse that he is having difficulty making decision
about terminating life support for his wife. What is the best initial action by the nurse?
A. Provide an opportunity for him to clarify his values related to the decision
B. Encourage him to share memories about his life with his wife and family
C. Advise him to seek several opinions before making decision
D. Offer to contact the hospital chaplain or social worker to offer support.
Answer: A. Provide an opportunity for him to clarify his values related to the decision
Rationale:
When a client is faced with a decisional conflict, the nurse should first provide opportunities
for the client to clarify values important in the decision. The rest may also be beneficial once
the client as clarified the values that are important to him in the decision-making process.

228. A client is being discharged home after being treated for heart failure (HF). What
instruction should the nurse include in this client’s discharge teaching plan?
A. Weigh every morning
B. Eat a high protein diet
C. Perform range of motion exercises
D. Limit fluid intake to 1,500 ml daily
Answer: A. Weigh every morning
229. A woman just learned that she was infected with Heliobacter pylori. Based on this
finding, which health promotion practice should the nurse suggest?
Answer: Encourage screening for a peptic ulcer
230. A client who recently underwear a tracheostomy is being prepared for discharge to
home. Which instructions is most important for the nurse to include in the discharge plan?
Answer: Teach tracheal suctioning techniques
231. A child with heart failure is receiving the diuretic furosemide (Lasix) and has serum
potassium level 3.0 mEq/L. Which assessment is most important for the nurse to obtain?
A. Cardiac rhythm and heart rate.
B. Daily intake of foods rich in potassium.
C. Hourly urinary output
D. Thirst ad skin turgor.
Answer: A. Cardiac rhythm and heart rate.
232. The nurse note a depressed female client has been more withdrawn and
noncommunicative during the past two weeks. Which intervention is most important to
include in the updated plan of care for this client?
A. Encourage the client’s family to visit more often
B. Schedule a daily conference with the social worker
C. Encourage the client to participate in group activities
D. Engage the client in a non-threatening conversation.
Answer: D. Engage the client in a non-threatening conversation.
Rationale:

Consistent attempts to draw the client into conversations which focus on non-threatening
subjects can be an effective means of eliciting a response, thereby decreasing isolation
behaviours. There is not sufficient data to support the effectiveness of A as an intervention for
this client. Although B may be indicated, nursing interventions can also be used to treat this
client. C is too threatening to this client.
233. A client with rheumatoid arthritis (RA) starts a new prescription of etanercept (Enbrel)
subcutaneously once weekly. The nurse should emphasize the importance of reporting
problem to the healthcare provider?
A. Headache
B. Joint stiffness
C. Persistent fever
D. Increase hunger and thirst
Answer: C. Persistent fever
Rationale:
Enbrel decrease immune and inflammatory responses, increasing the client’s risk of serious
infection, so the client should be instructed to report a persistent fever, or other signs of
infection to the healthcare provider.
234. The nurse is assessing an older adult with type 2 diabetes mellitus. Which assessment
finding indicates that the client understands long- term control of diabetes?
A. The fating blood sugar was 120 mg/dl this morning.
B. Urine ketones have been negative for the past 6 months
C. The haemoglobin A1C was 6.5g/100 ml last week
D. No diabetic ketoacidosis has occurred in 6 months.
Answer: C. The haemoglobin A1C was 6.5g/100 ml last week
Rationale:
A haemoglobin A1C level reflects he average blood sugar the client had over the previous 2
to 3 month, and level of 6.5 g/100 ml suggest that the client understand long-term diabetes
control. Normal value in a diabetic patient is up to 6.5 g/100 ml.
235. An older male client is admitted with the medical diagnosis of possible cerebral vascular
accident (CVA). He has facial paralysis and cannot move his left side. When entering the

room, the nurse finds the client’s wife tearful and trying unsuccessfully to give him a drink of
water. What action should the nurse take?
Answer: Ask the wife to stop and assess the client’s swallowing reflex
236. A 13 years-old client with non-union of a comminated fracture of the tibia is admitted
with osteomyelitis. The healthcare provider collects home aspirate specimens for culture and
sensitivity and applies a cast to the adolescent’s lower leg. What action should the nurse
implement next?
A. Administer antiemetic agents
B. Bivalve the cast for distal compromise
C. Provide high- calorie, high-protein diet
D. Begin parenteral antibiotic therapy
Answer: D. Begin parenteral antibiotic therapy
Rationale:
The standard of treatment for osteomyelitis is antibiotic therapy and immobilization. After
bond and blood aspirate specimens are obtained for culture and sensitivity, the nurse should
initiate parenteral antibiotics as prescribed.
237. The nurse is preparing a community education program on osteoporosis. Which
instruction is helpful in preventing bone loss and promoting bone formation?
Answer: Recommend weigh bearing physical activity
238. A client with a history of chronic pain requests a nonopioid analgesic. The client is alert
but has difficulty describing the exact nature and location of the pain to the nurse. What
action should the nurse implement next?
Answer: Administer the analgesic as requested
239. A male client receives a thrombolytic medication following a myocardial infarction.
When the client has a bowel movement, what action should the nurse implement?
A. Send stool sample to the lab for a guaiac test
B. Observe stool for a day-coloured appearance.
C. Obtain specimen for culture and sensitivity analysis
D. Asses for fatty yellow streaks in the client’s stool.
Answer: A. Send stool sample to the lab for a guaiac test

Rationale:
Thrombolytic drugs increase the tendency for bleeding. So guaiac (occult blood test) test of
the stool should be evaluated to detect bleeding in the intestinal tract.
240. The mother of a child with cerebral palsy (CP) ask the nurse if her child’s impaired
movements will worsen as the child grows. Which response provides the best explanation?
Answer: Brain damage with CP is not progressive but does have a variable course
241. During shift report, the central electrocardiogram (EKG) monitoring system alarms.
Which client alarm should the nurse investigate first?
Answer: Respiratory apnea of 30 seconds
242. In early septic shock states, what is the primary cause of hypotension?
A. Peripheral vasoconstriction
B. Peripheral vasodilation
C. Cardiac failure
D. A vagal response
Answer: B. Peripheral vasodilation
Rationale:
Toxins released by bacteria in septic shock create massive peripheral vasodilation and
increase microvascular permeability at the site of the bacterial invasion.
243. A client diagnosed with calcium kidney stones has a history of gout. A new prescription
for aluminium hydroxide (Amphogel) is scheduled to begin at 0730. Which client medication
should the nurse bring to the healthcare provider’s attention?
A. Allopurinol (Zyloprim)
B. Aspirin, low dose
C. Furosemide (lasix)
D. Enalapril (vasote)
Answer: A. Allopurinol (Zyloprim)
244. A male client’s laboratory results include a platelet count of 105,000/ mm 3 Based on this
finding the nurse should include which action in the client’s plan of care?
A. Cluster care to conserve energy

B. initiate contact isolation
C. Encourage him to use an electric razor
D. Asses him for adventitious lung sounds
Answer: C. Encourage him to use an electric razor
Rationale:
This client is at risk for bleeding based on his platelet count (normal 150,000 to 400,000/
mm3). Safe practices, such as using an electric razor for shaving, should be encouraged to
reduce the risk of bleeding.
245. A client is admitted to the hospital after experiencing a brain attack, commonly referred
to as a stroke or cerebral vascular accident (CVA). The nurse should request a referral for
speech therapy if the client exhibits which finding?
A. Abnormal responses for cranial nerves I and II
B. Persistent coughing while drinking
C. Unilateral facial drooping
D. Inappropriate or exaggerated mood swings
Answer: B. Persistent coughing while drinking
246. At 1615, prior to ambulating a postoperative client for the first time, the nurse reviews
the client’s medical record. Based on date contained in the record, what action should the
nurse take before assisting the client with ambulation:
A. Remove sequential compression devices.
B. Apply PRN oxygen per nasal cannula.
C. Administer a PRN dose of an antipyretic.
D. Reinforce the surgical wound dressing.
Answer: A. Remove sequential compression devices.
Rationale:
Sequential compression devices should be removed prior to ambulation and there is no
indication that this action is contraindicated. The client’s oxygen saturation levels have been
within normal limits for the previous four hours, so supplemental oxygen is not warranted.
247. Which assessment finding for a client who is experiencing pontine myelinolysis should
the nurse report to the healthcare provider?
A. Sudden dysphagia

B. Blurred visual field
C. Gradual weakness
D. Profuse diarrhoea
Answer: A. Sudden dysphagia
248. A client is scheduled to receive an IW dose of ondansetron (Zofran) eight hours after
receiving chemotherapy. The client has saline lock and is sleeping quietly without any
restlessness. The nurse caring for the client is not certified in chemotherapy administration.
What action should the nurse take?
A. Ask a chemotherapy-certified nurse to administer the Zofran
B. Administer the Zofran after flushing the saline lock with saline
C. Hold the scheduled dose of Zofran until the client awakens
D. Awaken the client to assess the need for administration of the Zofran.
Answer: B. Administer the Zofran after flushing the saline lock with saline
Rationale:
Zofran is an antiemetic administered before and after chemotherapy to prevent vomiting. The
nurse should administer the antiemetic using the accepter technique for IV administration via
saline lock. Zofran is not a chemotherapy drug and does not need to be administered by a
chemotherapy- certified nurse.
249. When providing diet teaching for a client with cholecystitis, which types of food choices
the nurse recommend to the client?
A. High protein
B. Low fat
C. Low sodium
D. High carbohydrate.
Answer: B. Low fat
Rationale:
A client with cholecystitis is at risk of gall stones that can be move into the biliary tract and
cause pain or obstruction. Reducing dietary fat decrease stimulation of the gall bladder, so
bile can be expelled, along with possible stones, into the biliary tract and small intestine.
250. A client with a history of cirrhosis and alcoholism is admitted with severe dyspnea and
ascites. Which assessment finding warrants immediate intervention by the nurse?

A. Jaundice skin tone
B. Muffled heart sounds
C. Pitting peripheral edema
D. Bilateral scleral edema
Answer: B. Muffled heart sounds
Rationale:
Muffled heart sounds may indicative fluid build-up in the pericardium and is life- threatening.
The other one are signs of end stage liver disease related to alcoholism but are not
immediately life- threatening.
251. When entering a client’s room, the nurse discovers that the client is unresponsive and
pulseless. The nurse initiate CPR and Calls for assistance. Which action should the nurse take
next?
A. Prepare to administer atropine 0.4 mg IVP
B. Gather emergency tracheostomy equipment
C. Prepare to administer lidocaine at 100 mg IVP
D. Place cardiac monitor leads on the client’s chest.
Answer: D. Place cardiac monitor leads on the client’s chest.
Rationale:
Before further interventions can be done, the client’s heart rhythm must be determined. This
can be done by connecting the client to the monitor. A or C are not a first line drug given for
any of the life threatening, pulses dysrhythmias
252. A client with a history of dementia has become increasingly confused at night and is
picking at an abdominal surgical dressing and the tape securing the intravenous (IV) line. The
abdominal dressing is no longer occlusive, and the IV insertion site is pink. What intervention
should the nurse implement?
A. Replace the IV site with a smaller gauge.
B. Redress the abdominal incision
C. Leave the lights on in the room at night.
D. Apply soft bilateral wrist restraints.
Answer: B. Redress the abdominal incision
Rationale:

The abdominal incision should be redressed using aseptic-techniques. The IV site should be
assessed to ensure that it has not been dislodged and a dressing reapplied, if need it. Leaving
the light on at night may interfere with the client’s sleep and increase confusion. Restraints
are not indicated and should only be used as a last resort to keep client from self-harm.
253. An adult male client is admitted to the emergency room following an automobile
collision in which he sustained a head injury. What assessment data would provide the
earliest that the client is experiencing increased intracranial pressure (ICP)?
A. Lethargy
B. Decorticate posturing
C. Fixed dilated pupil
D. Clear drainage from the ear.
Answer: A. Lethargy
Rationale:
Lethargy is the earliest sign of ICP along with slowing of speech and response to verbal
commands. The most important indicator of increase ICP is the client’s level or
responsiveness or consciousness. B and C are very late signs of ICP.
254. In preparing a diabetes education program, which goal should the nurse identify as the
primary emphasis for a class on diabetes self-management?
A. Prepare the client to independently treat their disease process
B. Reduce healthcare costs related to diabetic complications
C. Enable clients to become active participating in controlling the disease process
D. Increase client’s knowledge of the diabetic disease process and treatment options.
Answer: C. Enable clients to become active participating in controlling the disease process
Rationale:
The primary goal of diabetic self- management education is to enable the client to become an
active participant in the care and control of disease process, matching levels of selfmanagement to the abilities of the individual client. The goal is to place the client in a
cooperative or collaborative role with healthcare professional rather than (A)
255. To reduce staff nurse role ambiguity, which strategy should the nurse manager
implemented?
A. Confirm that all the staff nurses are being assigned to equal number of clients.

B. Review the staff nurse job description to ensure that it is clear, accurate, and recurrent.
C. Assign each staff nurse a turn unit charge nurse on a regular, rotating basis.
D. Analyze the amount of overtime needed by the nursing staff to complete assignments.
Answer: B. Review the staff nurse job description to ensure that it is clear, accurate, and
recurrent.
Rationale:
Role ambiguity occurs when there is inadequate explanation of job descriptions and assigned
tasks, as well as the rapid technological changes that produce uncertainty and frustration. A
and D may be implemented if the nurse manager is concerned about role overload, which is
the inability to accomplish the tasks related to one’s role. C is not related to ambiguity.
256. The nurse is assisting a new mother with infant feeding. Which information should the
nurse provide that is most likely to result in a decrease milk supply for the mother who is
breastfeeding?
A. Supplemental feedings with formula
B. Maternal diet high in protein
C. Maternal intake of increased oral fluid
D. Breastfeeding every 2 or 3 hours.
Answer: A. Supplemental feedings with formula
Rationale:
Infant sucking at the breast increases prolactin release and proceeds a feedback mechanism
for the production of milk, the nurse should explain that supplemental bottle formula feeding
minimizes the infant’s time at the breast and decreases milk supply. B promotes milk
production and healing after delivery. C support milk production. C is recommended routine
for breast feeding that promote adequate milk supply.
257. Which assessment is more important for the nurse to include in the daily plan of care for
a client with a burned extremity?
A. Range of Motion
B. Distal pulse intensity
C. Extremity sensation
D. Presence of exudate
Answer: B. Distal pulse intensity
Rationale:

Distal pulse intensity assesses the blood flow through the extremity and is the most important
assessment because it provides information about adequate circulation to the extremity.
Range of motions evaluates the possibility of long term contractures sensation. C evaluates
neurological involvement, and exudate. D provides information about wound infection, but
this assessment do not have the priority of determining perfusion to the extremity.
258. An elderly client with degenerative joint disease asks if she should use the rubber jar
openers that are available. The nurse’s response should be based on which information about
assistive devices?
Answer: They decrease the risk for joint trauma
259. When assessing a 6-month old infant, the nurse determines that the anterior fontanel is
bulging. In which situation would this finding be most significant?
A. Crying
B. Straining on stool
C. Vomiting
D. Sitting upright.
Answer: D. Sitting upright.
Rationale:
The anterior fontanel closes at 9 months of age and may bulge when venous return is reduced
from the head, but a bulging anterior fontanel is most significant if the infant is sitting up and
may indicated an increase in cerebrospinal fluid. Activities that reduce venous return from the
head, such as crying, a Valsalva maneuver, vomiting or a dependent position of the head,
cause a normal transient increase in intracranial pressure.
260. A client with angina pectoris is being discharge from the hospital. What instruction
should the nurse plan to include in this discharge teaching?
A. Engage in physical exercise immediately after eating to help decrease cholesterol levels.
B. Walk briskly in cold weather to increase cardiac output
C. Keep nitroglycerin in a light-colored plastic bottle and readily available.
D. Avoid all isometric exercises, but walk regularly.
Answer: D. Avoid all isometric exercises, but walk regularly.
Rationale:

Isometric exercise can raise blood pressure for the duration of the exercise, which may be
dangerous for a client with cardiovascular disease, while walking provides aerobic
conditioning that improves ling, blood vessel, and muscle function. Client with angina should
refrain from physical exercise for 2 hours after meals, but exercising does not decrease
cholesterol levels. Cold water cause vasoconstriction that may cause chest pain. Nitroglycerine should be readily available and stored in a dark-coloured glass bottle not C, to
ensure freshness of the medication.
261. What is the priority nursing action when initiating morphine therapy via an intravenous
patient-controlled analgesia (PCA) pump?
A. Initiate the dosage lockout mechanism on the PCA pump
B. Instruct the client to use the medication before the pain becomes severe
C. Assess the abdomen for bowel sounds.
D. Assess the client ability to use a numeric pain scale
Answer: A. Initiate the dosage lockout mechanism on the PCA pump
262. While undergoing haemodialysis, a male client suddenly complains of dizziness. He is
alert and oriented, but his skin is cool and clammy. His vital signs are: heart rate 128
beats/minute, respirations 18 breaths/minute, and blood pressure 90/60. Which intervention
should the nurse implement first?
Answer: Raise the client’s legs and feet
263. The nurse receives a newborn within the first minutes after a vaginal delivery and
intervenes to establish adequate respirations. What priority issue should the nurse address to
ensure the newborn’s survival?
A. Heat loss
B. Hypoglycemia
C. Fluid balance
D. Bleeding tendencies
Answer: A. Heat loss
264. The fire alarm goes off while the charge nurse is receiving the shift report. What action
should the charge nurse implement first?
Answer: Tell the staff to keep all clients and visitors in the client rooms with the doors closed

265. A 60-year-old female client asks the nurse about hormones replacement therapy (HRT)
as a means preventing osteoporosis. Which factor in the client’s history is a possible
contraindication for the use of HRT?
Answer: Her mother and sister have a history of breast cancer
266. A male client, who is 24 hours postoperative for an exploratory laparotomy, complains
that he is “starving” because he has had no “real food” since before the surgery. Prior to
advancing his diet, which intervention should the nurse implement?
Answer: Auscultate bowel sounds in all four quadrants
267. The nurse working in the psychiatric clinic has phone messages from several clients.
Which call should the nurse return first?
Answer: A family member of a client with dementia who has been missing for five hours
268. During change of shift, the nurse reports that a male client who had abdominal surgery
yesterday increasingly confused and disoriented during the night. He wandered into other
clients rooms, saying that there are men in his room trying to hurt him. Because of continuing
disorientation and the client’s multiple attempts to get of bed, soft restrains were applied at
0400. In what order should the nurse who is receiving report implement these interventions?
(Arrange from first action on top to last on the bottom).
1. Assess the client’s skin and circulation for impairment related to the restrains
2. Evaluate the client’s mentation to determine need to continue the restrains
3. Assign unlicensed assistive personnel to remove restrains and remain with client
4. Contact the client’s surgeon and primary healthcare provider
Answer: 2. Evaluate the client’s mentation to determine need to continue the restrains
1. Assess the client’s skin and circulation for impairment related to the restrains
3. Assign unlicensed assistive personnel to remove restrains and remain with client
4. Contact the client’s surgeon and primary healthcare provider
269. A mother brings her 3-year-old son to the emergency room and tells the nurse the he has
had an upper respiratory infection for the past two days. Assessment of the child reveals a
rectal temperature of 102 F. he is drooling and becoming increasingly more restless. What
action should the nurse take first?

Answer: Notify the healthcare provider and obtain a tracheostomy tray
270. After receiving the first dose of penicillin, the client begins wheezing and has trouble
breathing. The nurse notifies the healthcare provider immediately and received several
prescriptions. Which medication prescription should the nurse administer first?
Answer: Epinephrine Injection, USP IV
271. Two clients ring their call bells simultaneously requesting pain medication. What action
should the nurse implement first?
Answer: Evaluate both client’s pain using a standardized pain scale
272. A client receives a new prescription for simvastatin (Zocor) 5 mg PO daily at bedtime.
What action should the nurse take?
Answer: Administer the medication as prescribed with a glass of water
273. Which client should the nurse assess frequently because of the risk for overflow
incontinence? A client
Answer: Who is confused and frequently forgets to go to the bathroom
274. While monitoring a client during a seizure, which interventions should the nurse
implement? (Select all that apply)
A. Move obstacle away from client
B. Monitor physical movements
C. Observe for a patent airway
D. Record the duration of the seizure
Answer: A. Move obstacle away from client
B. Monitor physical movements
C. Observe for a patent airway
D. Record the duration of the seizure
275. A male client with a long history of alcoholism is admitted because of mild confusion
and fine motor tremors. He reports that he quit drinking alcohol and stopped smoking
cigarettes one month ago after his brother died of lung cancer. Which intervention is most
important for the nurses to include in the client’s plan of care?

A. Determine client’s level current blood alcohol level.
B. Observe for changes in level of consciousness.
C. Involve the client’s family in healthcare decisions.
D. Provide grief counseling for client and his family.
Answer: B. Observe for changes in level of consciousness.
Rationale:
Based on the client’s history of drinking, he may be exhibiting sing of hepatic involvement
and encephalopathy. Changes in the client’s level of consciousness should be monitored to
determine if he able to maintain consciousness, so neurological assessment has the highest
priority.
276. An older adult female admitted to the intensive care unit (ICU) with a possible stroke is
intubated with ventilator setting of tidal volume 600, PlO2 40%, and respiratory rate of 12
breaths/minute. The arterial blood gas (ABG) results after intubation are PH 7.31. PaCO 2 60,
PaO2 104, SPO2 98%, HCO3 23. To normalize the client’s ABG finding, which action is
required?
A. Report the results to the healthcare provider.
B. Increase ventilator rate.
C. Administer a dose of sodium carbonate.
D. Decrease the flow rate of oxygen.
Answer: B. Increase ventilator rate.
Rationale:
This client is experiencing respiratory acidosis. Increasing the ventilator rate depletes CO 2 a,
which returns the PH toward normal. Report findings is important but only after increasing
ventilator rate.
277. The mother of the 12- month-old with cystic fibrosis reports that her child is
experiencing increasing congestion despite the use of chest physical therapy (CPT) twice a
day, and has also experiences a loss of appetite. What instruction should the nurse provide?
A. Perform CPT after meals to increase appetite and improve food intake.
B. CPT should be performed more frequently, but at least an hour before meals.
C. Stop using CPT during the daytime until the child has regained an appetite.
D. Perform CPT only in the morning, but increase frequency when appetite improves.
Answer: B. CPT should be performed more frequently, but at least an hour before meals.

Rationale:
CPY with inhalation therapy should be performed several times a day to loosen the secretions
and move them from the peripheral airway into the central airways where they can be
expectorated. CPT should be done at least one hour before meals or two hours after meals.
278. The nurse is evaluating the diet teaching of a client with hypertension. What dinner
selection indicates that the client understands the dietary recommendation for hypertension?
Answer: Baked pork chop, applesauce, corn on the cob, 2% milk, and key-lime pie
279. A client with type 2 diabetes mellitus is admitted for frequent hyperglycemic episodes
and a glycosylated haemoglobin (HbA1c) of 10%. Insulin glargine 10 units subcutaneously
once a day at bedtime and a sliding scale with insulin aspart q6h are prescribed. What action
should the nurse include in this client’s plan of care?
A. Fingerstick glucose assessment q6h with meals
B. Mix bedtime dose of insulin glargine with insulin aspart sliding scale dose
C. Review with the client proper foot care and prevention of injury
D. Do not contaminate the insulin aspart so that it is available for iv use
E. Coordinate carbohydrate controlled meals at consistent times and intervals
F. Teach subcutaneous injection technique, site rotation and insulin management
Answer: A. Fingerstick glucose assessment q6h with meals
C. Review with the client proper foot care and prevention of injury
E. Coordinate carbohydrate controlled meals at consistent times and intervals
F. Teach subcutaneous injection technique, site rotation and insulin management
280. Which problem reported by a client taking lovastatin requires the most immediate fallow
up by the nurse?
A. Diarrhoea and flatulence
B. Abdominal cramps
C. Muscle pain
D. Altered taste
Answer: C. Muscle pain
Rationale:

Statins can cause rhabdomyolysis, a potentially fatal disease of skeletal muscle characterized
by myoglobinuria and manifested with muscle pain, so this symptom should immediately be
reported to the HCP.
281. While assessing a client’s chest tube (CT), the nurse discovers bubbling in the water seal
chamber of the chest tube collection device. The client’s vital signs are: blood pressure of
80/40 mmHg, heart rate 120 beats/minutes, respiratory rate 32 breaths/minutes, oxygen
saturation 88%. Which interventions should the nurse implement?
A. Provide supplemental oxygen
B. Auscultate bilateral lung fields
C. Administer a nebulizer treatment
D. Reinforce occlusive CT dressing
E. Give PRN dose of pain medication
Answer: A. Provide supplemental oxygen
B. Auscultate bilateral lung fields
D. Reinforce occlusive CT dressing
Rationale:
The air bubbles indicate an air leak from the lungs, the chest tube site, or the chest tube
collection system. Providing oxygen improves the oxygen saturation until the leak has been
resolved. Auscultating the lung fields helps to identify absent or decrease lung sound due to
collapsing lung.
282. Before leaving the room of a confused client, the nurse notes that a half bow knot was
used to attach the client's wrist restraints to the movable portion of the client's bed frame.
What action should the nurse take before leaving the room?
A. Ensure that the knot can be quickly released.
B. Tie the knot with a double turn or square knot.
C. Move the ties so the restraints are secured to the side rails.
D. Ensure that the restraints are snug against the client's wrist.
Answer: A. Ensure that the knot can be quickly released.
283. Oral antibiotics are prescribed for an 18-month-old toddler with severe otitis media. An
antipyrine and benzocaine-otic also prescribed for pain and inflammation. What instruction

should the nurse emphasize concerning the installation of the antipyrine/benzocaine otic
solution?
A. Place the dropper on the upper outer ear canal and instill the medication slowly.
B. Warm the medication in the microwave for 10 seconds before instilling.
C. Keep the medication refrigerated between administrations.
D. Have the child lie with the ear up for one to two minute after installation.
Answer: D. Have the child lie with the ear up for one to two minute after installation.
284. An older adult male is admitted with complications related to chronic obstructive
pulmonary disease (COPD). He reports progressive dyspnea that worsens on exertion and his
weakness has increased over the past month. The nurse notes that he has dependent edema in
both lower legs. Based on these assessment findings, which dietary instruction should the
nurse provide?
A. Limit the intake of high calorie foods.
B. Eat meals at the same time daily.
C. Maintain a low protein diet.
D. Restrict daily fluid intake.
Answer: D. Restrict daily fluid intake.
Rationale:
The client is exhibiting signs of cor pulmonale, a complication of COPD that causes the right
side of the heart to fail. Restricting fluid intake to 1000 to 2000 ml/day, eating a high-calorie
diet at small frequent meals with foods that are high in protein and low in sodium can help
relive the edema and decrease workload on the right-side of the heart.
285. The nurse inserts an indwelling urinary catheter as seen in the video what action should
the nurse take next?
A. Remove the catheter and insert into urethral opening
B. Observe for urine flow and then inflate the balloon.
C. Insert the catheter further and observe for discomfort.
D. Leave the catheter in place and obtain a sterile catheter.
Answer: D. Leave the catheter in place and obtain a sterile catheter.
Rationale:
The catheter is in the vaginal opening.

286. A client with coronary artery disease who is experiencing syncopal episodes is admitted
for an electrophysiology study (EPS) and possible cardiac ablation therapy. Which
intervention should the nurse delegate to the unlicensed assistive personnel (UAP)?
A. Prepare the skin for procedure.
B. Identify client's pulse points
C. Witness consent for procedure
D. Check telemetry monitoring
Answer: A. Prepare the skin for procedure.
287. Fallowing an outbreak of measles involving 5 students in an elementary school, which
action is most important for the school nurse to take?
A. Review the immunization records of all children in the elementary school
B. Report the measles outbreak to all community health organizations
C. Schedule a mobile public health vehicle to offer measles inoculations to unvaccinated
children.
D. Restrict unvaccinated children from attending school until measles outbreak is resolved.
Answer: D. Restrict unvaccinated children from attending school until measles outbreak is
resolved.
288. A preeclamptic client who delivered 24h ago remains in the labor and delivery recovery
room. She continues to receive magnesium sulphate at 2 grams per hour. Her total input is
limited to 125 ml per hour, and her urinary output for the last hour was 850 ml. What
intervention should the nurse implement?
A. discontinue the magnesium sulphate immediately
B. Decrease the client's iv rate to 50 ml per hour
C. Continue with the plan of care for this client
D. Change the client's to NPO status
Answer: C. Continue with the plan of care for this client
Rationale:
Continue with the plan. Diuresis in 24 to 48h after birth is a sign of improvement in the
preeclamptic client. As relaxation of arteriolar spasms occurs, kidney perfusion increases.
With improvement perfusion, fluid is drawn into the intravascular bed from the interstitial
tissue and then cleared by the kidneys

289. The nurse is planning care for a client who admits having suicidal thoughts. Which
client behavior indicates the highest risk for the client acting on these suicidal thoughts?
A. Express feelings of sadness and loneliness
B. Neglects personal hygiene and has no appetite
C. Lacks interest in the activity of the family and friends
D. Begin to show signs of improvement in affect
Answer: D. Begin to show signs of improvement in affect
Rationale:
When a depressed client begins to show signs of improvement, it can be because the client
has "figured out" how to be successful in committing suicide. Depressed clients, particularly
those who have shown signs of potentially becoming suicidal, should be watched with care
for an impending suicide attempt might be greater when the client appear suddenly happy,
begin to give away possessions, or becomes more relaxed and talkative.
290. When assessing a multigravida the first postpartum day, the nurse finds a moderate
amount of lochia rubra, with the uterus firm, and three fingerbreadths above the umbilicus.
What action should the nurse implement first?
A. Massage the uterus to decrease atony
B. Check for a destined bladder
C. Increase intravenous infusion
D. Review the haemoglobin to determined haemorrhage
Answer: B. Check for a destined bladder
Rationale:
A fundus that is extroverted (up to the right) and elevated above the umbilicus is indicative of
bladder distension/urine retention.
291. A 12 year old client who had an appendectomy two days ago is receiving 0.9% normal
saline at 50 ml/hour. The client's urine specific gravity is 1.035. What action should the nurse
implement?
A. Evaluate postural blood pressure measurements
B. Obtain specimen for uranalysis
C. Encourage popsicles and fluids of choice
D. Assess bowel sounds in all quadrants
Answer: C. Encourage popsicles and fluids of choice

Rationale:
Specific gravity of urine is a measurement of hydration status (normal range of 1.010 to
1.025) which is indicative of fluid volume deficit when Sp Gr increases as urine becomes
more concentrated.
292. An older male client arrives at the clinic complaining that his bladder always feels full.
He complains of weak urine flow, frequent dribbling after voiding, and increasing nocturia
with difficulty initiating his urine stream. Which action should the nurse implement?
A. Obtain a urine specimen for culture and sensitivity
B. Palpate the client's suprapubic area for distention
C. Advise the client to maintain a voiding diary for one week
D. Instruct in effective technique to cleanse the glans penis
Answer: B. Palpate the client's suprapubic area for distention
Rationale:
The client is exhibiting classic signs of an enlarge prostate gland, which restricts urine flow
and cause bothersome lower urinary tract symptoms (LUTS) and urinary retention, which is
characterized by the client's voiding patterns and perception of incomplete bladder emptying.
293. The nurse is preparing to administer 1.6 ml of medication IM to a 4 month old infant.
Which action should the nurse include?
A. Select a 22 gauge 1 ½ inch (3.8 cm) needle for the intramuscular injection
B. Administer into the deltoid muscle while the parent holds the infant securely
C. Divide the medication into two injection with volumes under 1ml
D. Use a quick dart-like motion to inject into the dorsogluteal site.
Answer: C. Divide the medication into two injection with volumes under 1ml
Rationale:
IM injection for children under 3 of age should not exceed 1ml. divide the dose into smaller
volumes for injection in two different sites.
294. A client who had a below the knee amputation is experiencing severe phantom limb pain
(PLP) and ask the nurse if mirror therapy will make the pain stop. Which response by the
nurse is likely to be most helpful?
A. Research indicates that mirror therapy is effective in reducing phantom limb pain
B. You can try mirror therapy, but do not expect to complete elimination of the pain

C. Transcutaneous electrical nerve stimulators (TENS) have been found to be more effective
D. Where did you learn about the use of mirror therapy in treating in treating phantom limb
pain?
Answer: A. Research indicates that mirror therapy is effective in reducing phantom limb pain
Rationale:
Pain relief associated with mirror therapy may be due to the activation of neurons in the
hemisphere of the brain that is contralateral to the amputated limb when visual input reduces
the activity of systems that perceive protopathic pain.
295. An older adult client with heart failure (HF) develops cardiac tamponade. The client has
muffled, distant, heart sounds, and is anxious and restless. After initiating oxygen therapy and
IV hydration, which intervention is most important for the nurse to implement?
A. Observe neck for jugular vein distention
B. Notify healthcare provider to prepare for pericardiocentesis
C. Asses for paradoxical blood pressure
D. Monitor oxygen saturation (SpO2) via continuous pulse oximetry
Answer: B. Notify healthcare provider to prepare for pericardiocentesis
Rationale:
Cardiac tamponade is pressure on the heart that occurs when blood or fluid builds up in the
space between the heart muscle (myocardium) and the outer covering sac of the heart
(pericardium). In this condition, blood or fluid collects in the pericardium, the sac
surrounding the heart. This prevents the heart ventricles from expanding fully. The excess
pressure from the fluid prevents the heart from working properly. As a result, the body does
not get enough blood.
296. A new member joins the nursing team spreads books on the table, puts items on two
chairs, and sits on a third chair. The members of the group are forced to move closer and
remove their possessions from the table what action should the nurse leader take?
A. Move to welcome and accommodate a new person
B. Ask the new person to move belonging to accommodate others
C. Tell the new person to move belongings because of limited space
D. Bring in additional chairs so that all staff members can be seated
Answer: B. Ask the new person to move belonging to accommodate others

297. The nurse is caring for a one week old infant who has a ventriculoperitoneal (VP) shunt
that was placed 2 days after birth. Which findings are an indication of a postoperative
complication?
A. Poor feeding and vomiting
B. Leakage of CSF from the incisional site
C. Hyperactive bowel sound
D. Abdominal distention
E. WBC count of 10000/mm3
Answer: A. Poor feeding and vomiting
B. Leakage of CSF from the incisional site
D. Abdominal distention
298. The nurse is preparing a heparin bolus dose of 80 units/kg for a client who weighs 220
pounds. Heparin sodium injection, USP is available in a 3o ml multidose vial with the
concentration of 1,000 USP units/ml. how many ml of heparin should the nurse administer?
(Enter numeric value only)
Answer: 8
Calculate the client’s weigh in kg: 220 pounds divides by 2.2 pounds/kg ꞊ 100 kg
Calculate the client’s dose, 80 units x 100 kg ꞊ 8,000 units Use the formula, D / H X Q ꞊
8,000 units / 1,000 units x 1ml ꞊ 8
299. In monitoring tissue perfusion in a client following an above the knee amputation (aka),
which action should the nurse include in the plan of care?
A. Evaluate closet proximal pulse.
B. Asses skin elasticity of the stump.
C. Observe for swelling around the stump.
D. Note amount color of wound drainage.
Answer: A. Evaluate closet proximal pulse.
Rationale:
A primary focus of care for a client with an AKA is monitoring for signs of adequate tissue
perfusion, which include evaluating skin color and ongoing assessment of pulse strength.

300. The leg of a client who is receiving hospice care have become mottled in appearance.
When the nurse observes the unlicensed assistive personal (UAP) place a heating pad on the
mottled areas, what action should the nurse take?
A. Remove the heating pads and place a soft blanket over the client’s leg and feet.
B. Advise the UAP to observe the client’s skin while the heating pads are in place.
C. Elevate the client’s feet on a pillow and monitor the client’s pedal pulses frequently.
D. Instruct the UAP to reposition the heating pads to the sides of the legs and feet.
Answer: A. Remove the heating pads and place a soft blanket over the client’s leg and feet.
301. A client who underwent an uncomplicated gastric bypass surgery is having difficult with
diet management. What dietary instruction is most important for the nurse to explain to the
client?
A. Chew food slowly and thoroughly before attempting to swallow
B. Plan volume-controlled evenly-space meal thorough the day
C. Sip fluid slowly with each meal and between meals
D. Eliminate or reduce intake fatty and gas forming food
Answer: B. Plan volume-controlled evenly-space meal thorough the day
Rationale:
It is most important for the client to learn how to eat without damaging the surgical site and
to keep the digestive system from dumping the food instead of digesting it. Eating volumecontrol and evenly-space meals thorough the day allows the client to fill full, avoid binging,
and eliminate the possibility of eating too much one time. Chewing slowly and thoroughly
helps prevent over eating by allowing a filling of fullness to occur. Taking sips, rather than
large amounts of fluids keeps the stomach from overfilling and allow for adequate calories to
be consumed. Gas forming foods and fatty foods should be avoiding to decrease risk of
dumping syndrome and flatulence.
302. If the nurse is initiating IV fluid replacement for a child who has dry, sticky mucous
membranes, flushed skin, and fever of 103.6 F. Laboratory finding indicate that the child has
a sodium concentration of 156 mEq/L. What physiologic mechanism contributes to this
finding?
A. The intravenous fluid replacement contains a hypertonic solution of sodium chloride
B. Urinary and Gastrointestinal fluid loss reduce blood viscosity and stimulate thirst
C. Insensible loss of body fluids contributes to the hemoconcentration of serum solutes

D. Hypothalamic resetting of core body temperature causes vasodilation to reduce body heat
Answer: C. Insensible loss of body fluids contributes to the hemoconcentration of serum
solutes
Rationale:
Fever causes insensible fluid loss, which contribute to fluid volume and results in
hemoconcentration of sodium (serum sodium greater than 150 mEq/L). Dehydration, which
is manifested by dry, sticky mucous membranes, and flushed skin, is often managed by
replacing lost fluids and electrolytes with IV fluids that contain varying concentration of
sodium chloride. Although other options are consistent with fluid volume deficit, the
physiologic response of hypernatremia is explained by hem concentration.
303. During a Woman’s Health fair, which assignment is the best for the Practical Nurse (PN)
who is working with a register nurse (RN)
A. Encourage the woman at risk for cancer to obtain colonoscopy.
B. Present a class of breast-self examination
C. Prepare a woman for a bone density screening
D. Explain the follow-up need it for a client with prehypertension.
Answer: C. Prepare a woman for a bone density screening
Rationale:
A bone density screening is a fast, noninvasive screening test for osteoporosis that can be
explained by the PN. There is no additional preparation needed (A) required a high level of
communication skill to provide teaching and address the client’s fear. (B) Requires a higher
level of client teaching skill than responding to one client. (D) Requires higher level of
knowledge and expertise to provide needed teaching regarding this complex topic.
304. An adult client present to the clinic with large draining ulcers on both lower legs that are
characteristics of Kaposi’s sarcoma lesions. The client is accompanied by two family
member. Which action should the nurse take?
A. Ask family member to wear gloves when touching the patient
B. Send family to the waiting area while the client’s history is taking
C. Obtain a blood sample to determine is the client is HIV positive
D. Complete the head to toes assessment to identify other sign of HIV
Answer: B. Send family to the waiting area while the client’s history is taking
Rationale:

To protect the client privacy, the family member should be asked to wait outside while the
client’s history is take. Gloves should be worn when touching the client’s body fluids if the
client is HIV positive and these lesion are actually Kaposi sarcoma lesion. HIV testing cannot
legally be done without the client explicit permission. A further assessment can be
implemented after the family left the room.
305. An adult client is exhibit the maniac stage of bipolar disorder is admitted to the
psychiatric unit. The client has lost 10 pounds in the last two weeks and has no bathed in a
week “I’m trying to start a new business and “I’m too busy to eat”. The client is oriented to
time, place, person but not situation. Which nursing problem has the greatest priority?
A. Hygiene-self-care deficit
B. Imbalance nutrition
C. Disturbed sleep pattern
D. Self-neglect
Answer: B. Imbalance nutrition
Rationale:
The client’s nutritional status has the highest priority at this time, and finger foods are often
provided, so the client who is on the maniac phase of bipolar disease can receive adequate
nutrition. Other options are nursing problems that should also be addresses with the client’s
plan of care, but at this stage in the client’s treatment, adequate nutrition is a priority
306. The nurse is preparing a discharge teaching plan for a client who had a liver transplant.
Which instruction is most important to include in this plan?
A. Limit intake fatty foods for one month after surgery.
B. Notify the healthcare provider if edema occurs.
C. Increase activity and exercise gradually, as tolerated.
D. Avoid crowds for first two months after surgery.
Answer: D. Avoid crowds for first two months after surgery.
Rationale:
Cyclosporine immunosuppression therapy is vital in the success of liver transplantation and
can increase the risk for infection, which is critical in the first two months after surgery. Fever
is often.

307. The nurse is assessing a client’s nailbeds. Witch appearance indicates further follow-up
is needed for problems associated with chronic hypoxia?
Answer:

308. A client who had a percutaneous transluminal coronary angioplasty (PTCA) two weeks
ago returns to the clinic for a follow up visit. The client has a postoperative ejection fraction
ejection fraction of 30%. Today the client has lungs which are clear, +1 pedal edema, and a
5pound weight gain. Which intervention the nurse implement?
A. Arrange transport for admission to the hospital.
B. Insert saline lock for IV diuretic therapy.
C. Assess compliance with routine prescriptions.
D. Instruct the client to monitor daily caloric intake.
Answer: C. Assess compliance with routine prescriptions.
Rationale:
Fluid retention may be a sign that the client is not taking the medication as prescribed or that
the prescriptions may need adjustment to manage cardiac function post-PTCA (normal
ejection fraction range is 50 to 75%)
309. The RN is assigned to care for four surgical clients. After receiving report, which client
should the nurse see first? The client who is
A. Two days postoperative bladder surgery with continuous bladder irrigation infusing.
B. One day postoperative laparoscopic cholecystectomy requesting pain medication.

C. Three days postoperative colon resection receiving transfusion of packed RBCs.
D. Preoperative, in buck’s traction, and scheduled for hip arthroplasty within the next 12
hours.
Answer: C. Three days postoperative colon resection receiving transfusion of packed RBCs.
310. The nurse is preparing an older client for discharge following cataract extraction.
Which instruction should be include in the discharge teaching?
A. Do not read without direct lighting for 6 weeks.
B. Avoid straining at stool, bending, or lifting heavy objects.
C. Irrigate conjunctiva with ophthalmic saline prior to installing antibiotic ointment.
D. Limit exposure to sunlight during the first 2 weeks when the cornea is healing.
Answer: B. Avoid straining at stool, bending, or lifting heavy objects.
Rationale:
After cataract surgery, the client should avoid activities which increase pressure and place
strain on the suture line.
311. The healthcare provider prescribes potassium chloride 25 mEq in 500 ml D_5W to
infuse over 6 hours. The available 20 ml vial of potassium chloride is labeled, “10 mEq/5ml.”
how many ml of potassium chloride should the nurse add the IV fluid? (Enter numeric value
only. If is rounding is required, round to the nearest tenth.)
Answer: 12.5
Rationale:
Using the formula D / H X Q: 25 mEq / 10 mEq x 5ml ꞊ 12.5ml
312. At 40 week gestation, a laboring client who is lying is a supine position tells the nurse
that she has finally found a comfortable position. What action should the nurse take?
A. Encourage the client to turn on her left side.
B. Place a pillow under the client’s head and knees.
C. Explain to the client that her position is not safe.
D. Place a wedge under the client’s right hip.
Answer: D. Place a wedge under the client’s right hip.
Rationale:

Hypotension from pressure on the vena cava is a risk for the full-term client. Placing a wedge
under the right hip will relieve pressure on the vena cava. Other options will either not relieve
pressure on the vena cava or would not allow the client the remaining her position of choice.
313. A client with a history of diabetes and coronary artery disease is admitted with shortness
of breath, anxiety, and confusion. The client’s blood pressure is 80/60 mmHg, heart rate 120
beats/minute with audible third and fourth heart sounds, and bibasilar crackles. The client’s
average urinary output is 5 ml/hour. Normal saline is infusing at 124 ml/hour with a
secondary infusion of dopamine at mcg/kg/minute per infusion pump. With intervention
should the nurse implement?
A. Irrigate the indwelling urinary catheter.
B. Prepare the client for external pacing.
C. Obtain capillary blood glucose measurement.
D. Titrate the dopamine infusion to raise the BP.
Answer: D. Titrate the dopamine infusion to raise the BP.
Rationale:
The client is experiencing cardiogenic shock and requires titration per protocol of the
vasoactive secondary infusion, dopamine, to increase the blood pressure. Low hourly urine
output is due to shock and does not indicate a need for catheter irrigation. Pacing is not
indicated based on the client’s capillary blood glucose should be monitored, but is not
directly indicated at this time.
314. The nurse ends the assessment of a client by performing a mental status exam.
Which statement correctly describes the purpose of the mental status exam?
A. Determine the client’s level of emotional functioning’
B. Assess functional ability of the primary support system.
C. Evaluate the client’s mood, cognition and orientation.
D. Review the client’s pattern of adaptive coping skill
Answer: C. Evaluate the client’s mood, cognition and orientation.
Rationale:
The mental status exam assesses the client for abnormalities in cognitive functioning;
potential thought processes, mood and reasoning, the other options listed are all components
of the client’s psychosocial assessment.

315. An older adult resident of a long-term care facility has a 5-year history of hypertension.
The client has a headache and rate the pain 5 on a pain scale 0 to 10. The client’s blood
pressure is currently 142/89. Which interventions should the nurse implement? (Select all that
apply)
A. Administer a daily dose of lisinopril as scheduled.
B. Assess the client for postural hypotension.
C. Notify the healthcare provider immediately
D. Provide a PRN dose of acetaminophen for headache
E. Withhold the next scheduled daily dose of warfarin.
Answer: A. Administer a daily dose of lisinopril as scheduled.
D. Provide a PRN dose of acetaminophen for headache
Rationale:
The client’ routinely scheduled medication, lisinopril, is an antihypertensive medication and
should be administered as scheduled to maintain the client’s blood pressure. A PRN dose of
acetaminophen should be given for the client’s headache. The other options are not indicated
for this situation.
316. When conducting diet teaching for a client who is on a postoperative soft diet, which
foods should eat? (Select all that apply)
A. Pasta, noodles, rice.
B. Egg, tofu, ground meat.
C. Mashed, potatoes, pudding, milk.
D. Brussel sprouts, blackberries, seeds.
E. Corn bran, whole wheat bread, whole grains.
Answer: A. Pasta, noodles, rice.
B. Egg, tofu, ground meat.
C. Mashed, potatoes, pudding, milk.
Rationale:
A client’s postoperative diet is commonly progressed as tolerated. A soft diet includes foods
that are mechanically soft in texture (pasta, egg, ground meat, potatoes, and pudding. High
fiber foods that require thorough chewing and gas forming foods, such as cruciferous
vegetables and fresh fruits with skin, grains and seeds are omitted.

317. The nurse is preparing a 4-day-old I infant with a serum bilirubin level of 19 mg/dl (325
micromol/L) for discharge from the hospital. When teaching the parents about home
phototherapy, which instruction should the nurse include in the discharge teaching plan?
A. Reposition the infant every 2 hours.
B. Perform diaper changes under the light.
C. Feed the infant every 4 hours.
D. Cover with a receiving blanket.
Answer: A. Reposition the infant every 2 hours.
Rationale:
An infant, who is receiving phototherapy for hyperbilirubinemia, should be repositioned
every two hours. The position changes ensure that the phototherapy lights reach all of the
body surface areas. Bathing, feedings, and diaper changes are ways for the parents to bond
with the infant, and can occur away from the treatment. Feedings need to occur more
frequently than every 4 hours to prevent dehydration. The infant should wear only a diaper so
that the skin is exposed to the phototherapy.
318. When planning care for a client with acute pancreatitis, which nursing intervention has
the highest priority?
A. Withhold food and fluid intake.
B. Initiate IV fluid replacement.
C. Administer antiemetic as needed.
D. Evaluate intake and output ratio.
Answer: A. Withhold food and fluid intake.
Rationale:
The pathophysiologic processes in acute pancreatitis result from oral fluid and ingestion that
causes secretion of pancreatic enzymes, which destroy ductal tissue and pancreatic cells,
resulting in auto digestion and fibrosis of the pancreas. The main focus of the nursing care is
reducing pain caused by pancreatic destruction through interventions that decrease GI
activity, such as keeping the client NPO. Other choices are also important intervention but are
secondary to pain management.
319. Assessment by the home health nurse of an older client who lives alone indicates that
client has chronic constipations. Daily medications include furosemide for hypertension and

heart failure and laxatives. To manage the client’s constipation, which suggestions should the
nurse provide? (Select all that apply)
A. Decrease laxative use to every other day, and use oil retention enemas as needed.
B. Include oatmeal with stewed pruned for breakfast as often as possible.
C. Increase fluid intake by keeping water glass next to recliner.
D. Recommend seeking help with regular shopping and meal preparation.
E. Report constipation to healthcare provider related to cardiac medication side effects.
Answer: B. Include oatmeal with stewed pruned for breakfast as often as possible.
C. Increase fluid intake by keeping water glass next to recliner.
D. Recommend seeking help with regular shopping and meal preparation.
Rationale:
Older adult are at higher risk for chronic constipation due to decreased gastrointestinal
muscle tone leading to reduce motility. Oatmeal with prunes increases dietary fiber and bowel
stimulation, thereby decreasing need for laxatives. Increased fluid intake also decreases
constipations. Assistance with food preparation might help the client eat more fresh fruits and
vegetables and result on less reliance on microwaved and fast foods, which are usually high
in sodium and fat with little fiber. Laxatives can be reduced gradually by improving the diet,
without resorting to using enemas.
320. A young boy who is in a chronic vegetative state and living at home is readmitted to the
hospital with pneumonia and pressure ulcers. The mother insists that she is capable of caring
for her son and which action should the nurse implement next?
A. Report the incident to the local child protective services.
B. Find a home health agency that specializes in brain injuries.
C. Determine the mother’s basic skill level in providing care.
D. Consult the ethics committee to determine how to proceed.
Answer: C. Determine the mother’s basic skill level in providing care.
Rationale:
Although the mother states she is a capable caregiver, the client is manifesting disuse
syndrome complications, and the mother’s skill in providing basic care should be determined.
Further assessment is needed before implementing other nursing actions.
321. After the risk and benefits of having a cardiac catheterization are reviewed by the
healthcare provider, an older adult with unstable angina is scheduled for the procedure. When

the nurse presents the consent form for signature, the client asks how the wires will keep a
heart heating during the procedure. What action should the nurse take?
A. Explain the procedure again in detail and clarify any misconceptions.
B. Notify the healthcare provider of the client’s lack of understanding.
C. Call the client’s next of kin and have them provide verbal consent.
D. Postpone the procedure until the client understands the risk and benefits.
Answer: B. Notify the healthcare provider of the client’s lack of understanding.
Rationale:
The nurse is only witnessing the signature, and is not responsible for the client’s
understanding of the procedure. The healthcare provider needs to clarify any questions and
misconceptions. Explaining the procedure again is the healthcare provider’s legal
responsibility. The other options are not indicated.
322. In assessing a client at 34-weeks’ gestation, the nurse notes that she has a slightly
elevated total T4 with a slightly enlarged thyroid, a hematocrit of 28%, a heart rate of 92
beats per minute, and a systolic murmur. Which finding requires follow-up?
A. Elevated thyroid hormone level.
B. Hematocrit of 28%.
C. Heart rate of 92 beats per minute.
D. Systolic murmur.
Answer: B. Hematocrit of 28%.
Rationale:
Although physiologic anemia is expected in pregnancy, a hematocrit of 28% is below
pregnant norms and could signify iron-deficiency anemia. Other options are normal finding
pregnancy
323. A client with osteoporosis related to long-term corticosteroid therapy receives a
prescription for calcium carbonate. Which client’s serum laboratory values requires
intervention by the nurse?
A. Total calcium 9 mg/dl (2.25 mmol/L SI)
B. Creatinine 4 mg/dl (354 micromol/L SI)
C. Phosphate 4 mg/dl (1.293 mmol/L SI)
D. Fasting glucose 95 mg/dl (5.3 mmol/L SI)
Answer: B. Creatinine 4 mg/dl (354 micromol/L SI)

324. A clinical trial is recommended for a client with metastatic breast cancer, but she refuses
to participate and tells her family that she does not wish to have further treatments. The
client’s son and daughter ask the nurse to try and convince their mother to reconsider this
decision. How should the nurse respond?
A. Ask the client with her children present if she fully understands the decision she has made.
B. Discuss success of clinical trials and ask the client to consider participating for one month.
C. Explain to the family that they must accept their mother’s decision.
D. Explore the client’s decision to refuse treatment and offer support
Answer: D. Explore the client’s decision to refuse treatment and offer support
Rationale:
As long as the client is alert, oriented and aware of the disease prognosis, the healthcare team
must abide by her decisions. Exploring the decision with the client and offering support
provides a therapeutic interaction and allows the client to express her fears and concerns
about her quality of life.
Other options are essentially arguing with the client’s decisions regarding her end of life
treatment or diminish the opportunity for the client to discuss her feelings
325. An adult client with severe depression was admitted to the psychiatric unit yesterday
evening. Although the client ran one year ago, his spouse states that the client no longer runs,
bur sits and watches television most of the day. Which is most important for the nurse to
include in this client’s plan of care for today?
A. Assist client in identifying goals for the day.
B. Encourage client to participate for one hour in a team sport.
C. Schedule client for a group that focuses on self-esteem.
D. Help client to develop a list of daily affirmations.
Answer: A. Assist client in identifying goals for the day.
Rationale:
Clients with severe depression have low energy and benefit from structured activities because
concentration is decreased. The client participate in care by identifying goals for the day is
the most important intervention for the client’s first day at the unit. Other options can be
implemented over time, as the depression decreases.

326. An adult who is 5 feet 5 inches (165.1 cm) tall and weighs 90 lb. (40.8 Kg) is admitted
with a diagnosis of chronic anorexia. The client receives a regular diet for 2 days, and the
client’s medical records indicates that 100% of the diet provided has been consumed.
However the client’s weight on the third day morning after admission is 89 lb. (40.4 Kg).
What action should the nurse implement?
A. Examine the client’s room for hidden food.
B. Assign staff to monitor what the client eats.
C. Ask the client if the food provided is being eaten or discarded.
D. Provide the client with a high calorie diet.
Answer: B. Assign staff to monitor what the client eats.
Rationale:
Clients with an eating disorder have an unhealthy obsession with food. The client’s continued
weight loss, despites indication that the client has consumed 100% of the diet, should raise
questions about the client’s intake of the food provided, so the client should be observed
during meals to prevent hiding or throwing away food. Other options may be accurate but
ineffective and unnecessary.
327. A client exposed to tuberculosis is scheduled to begin prophylactic treatment with
isoniazid. Which information is most important for the nurse to note before administering the
initial dose?
A. Conversion of the client’s PPD test from negative to positive.
B. Length of time of the exposure to tuberculosis.
C. Current diagnosis of hepatitis B.
D. History of intravenous drug abuse.
Answer: C. Current diagnosis of hepatitis B.
Rationale:
Prophylactic treatment of tuberculosis with isoniazid is contraindicated for persons with liver
disease because it may cause liver damage. The nurse should withhold the prescribed dose
and contact the healthcare provider. Other options do not provide data indicating the need to
question or withhold the prescribed treatment.
328. The nurse walks into a client’s room and notices bright red blood on the sheets and on
the floor by the IV pole. Which action should the nurse take first?
A. Clean up the spilled blood to reduce infection transmission.

B. Notify the healthcare provider that the client appears to be bleeding.
C. Apply direct pressure to the client’s IV site.
D. Identify the source and amount of bleeding.
Answer: D. Identify the source and amount of bleeding.
Rationale:
The nursed should first assess the client to determine the action that should be taken. Patient
safety is the priority; other options are not priority.
329. During a routine clinic visit, an older female adult tells the nurse that she is concerned
that the flu season is coming soon, but is reluctant to obtain the vaccination. What action
should the nurse take first?
A. Determine when the client last had an influenza vaccination.
B. Discuss the concerns expressed by the client about the vaccination.
C. Ask about any recent exposure to persons with the flu or other viruses.
D. Review the informed consent form for the vaccination with the client.
Answer: B. Discuss the concerns expressed by the client about the vaccination.
Rationale:
The nurse should first address the concerns identified by the client, before taking other
actions, such as obtaining information about past vaccinations, exposure to the flu, or
reviewing the informed consent form.
330. A client is admitted with acute pancreatitis. The client admits to drinking a pint of
bourbon daily. The nurse medicates the client for pain and monitors vital signs q2 hours.
Which finding should the nurse report immediately to the healthcare provider?
A. Confusion and tremors
B. Yellowing and itching of skin.
C. Abdominal pain and vomiting
D. Anorexia and abdominal distention
Answer: A. Confusion and tremors
Rationale:
Daily alcohol is the likely etiology for the client’s pancreatitis. Abrupt cessation of alcohol
can result in delirium tremens (DT) causing confusion and tremors, which can precipitate
cardiovascular complications and should be reported immediately to avoid life-threatening

complications. The other options are expected findings in those with liver dysfunction or
pancreatitis, but do not require immediate action.
331. The nurse is teaching a mother of a newborn with a cleft lip how to bottle feed her baby
using medela haberman feeder, which has a valve to control the release of milk and a slit
nipple opening. The nurse discusses placing the nipple’s elongated tip in the back of the oral
cavity. What instructions should the nurse provide the mother about feedings?
A. Squeeze the nipple base to introduce milk into the mouth
B. Position the baby in the left lateral position after feeding
C. Alternate milk with water during feeding
D. Hold the newborn in an upright position
Answer: D. Hold the newborn in an upright position
Rationale:
The mother should be instructed to hold the infant during feedings in a sitting or upright
position to prevent aspiration. Impaired sucking is compensated by the use of special feeding
appliances and nipples such as the haberman feeder that prevents aspiration by adjusting the
flow of mild according to the effort of the neonate. Squeezing the nipple base may introduce
a volume that is greater than the neonate can coordinate swallowing. The preferred position
of an infant after feeding is on the right side to facilitate stomach emptying. Sucking
difficulty impedes the neonate’s intake of adequate nutrient needed for weight gain and water
should be provided after the feeding to cleanse the oral cavity and not fill up the neonate’s
stomach.
332. Following and gunshot wound, an adult client a haemoglobin level of 4 grams/dl (40
mmol/L SI). The nurse prepares to administer a unit of blood for an emergency transfusion.
The client has AB negative blood type and the blood bank sends a unit of type A Rh negative,
reporting that there is not type AB negative blood currently available. Which intervention
should the nurse implement?
A. Transfuse Type A negative blood until type AB negative is available.
B. Recheck the client’s haemoglobin, blood type and Rh factor.
C. Administer normal saline solution until type AB negative is available
D. Obtain additional consent for administration of type A negative blood
Answer: A. Transfuse Type A negative blood until type AB negative is available.
Rationale:

Those who have type AB blood are considered universal recipients using A or B blood types
that is the same Rh factor. The client’s haemoglobin is critically low and the client should
receive a unit of blood that is type A, which must be Rh negative blood. Other options are not
indicated in this situation.
333. A young adult female college student visits the health clinic in early winter to obtain
birth control pills. The clinic nurse asks if the student has received an influenza vaccination.
The student stated she did not receive vaccination because she has asthma. How should the
nurse respond?
A. Offer to provide the influenza vaccination to the student while she is at the clinic
B. Encourage the student to obtain a vaccination prior to the next influenza season.
C. Confirm that a history of asthma can increase risks associated with the vaccine.
D. Advise the student that the nasal spray vaccine reduces side effects for people with
asthma.
Answer: A. Offer to provide the influenza vaccination to the student while she is at the clinic
Rationale:
Person with asthma are at increased risk related to influenza and should receive the influenza
vaccination prior to or during influenza season. Waiting until the start of the next season
places the student at risk for the current season. The vaccination does not increase risk for
persons with asthma but the nasal spray may result in increased wheezing after receiving that
form of the vaccination.
334. A client with eczema is experiencing severe pruritus. Which PRN prescriptions should
the nurse administer? (Select all that apply)
A. Topical corticosteroid.
B. Topical scabicide.
C. Topical alcohol rub.
D. Transdermal analgesic.
E. Oral antihistamine
Answer: A. Topical corticosteroid.
E. Oral antihistamine
Rationale:
Anti-inflammatory actions of topical corticosteroids and oral antihistamines provide relief
from severe pruritus (itching). Other options are not indicated.

335. The nurse is using a straight urinary catheter kit to collect a sterile urine specimen from
a female client. After positioning am prepping this client, rank the actions in the sequence
they should be implemented. (Place to first action on the top on the last action on the bottom.)
1. Open the sterile catheter kit close to the client’s perineum.
2. Don sterile gloves and prepare to sterile field
3. Cleanse the urinary meatus using the solution, swabs, and forceps provided
4. Place distal end of the catheter in sterile specimen cup and insert catheter into meatus
Answer: 1. Open the sterile catheter kit close to the client’s perineum.
2. Don sterile gloves and prepare to sterile field
3. Cleanse the urinary meatus using the solution, swabs, and forceps provided
4. Place distal end of the catheter in sterile specimen cup and insert catheter into meatus
Rationale:
First the kit should be open near the clients to minimize the risk of contamination during the
collection of the sterile specimen. Once the kit is opened, sterile gloves should be donned to
prepare the sterile field. Then the clients’ meatus should be cleansed, and the catheter inserted
while to distal end of the catheter drains urine into the sterile specimen cup or receptacle.
336. An adult male was diagnosed with stage IV lung cancer three weeks ago. His wife
approaches the nurse and asks how she will know that her husband's death is imminent
because their two adult children want to be there when he dies. What is the best response by
the nurse?
A. Explain that the client will start to lose consciousness and his body system will slow down
B. Reassure the spouse that the healthcare provider will let her know when to call the
children
C. Offer to discuss the client’s health status with each of the adult children
D. Gather information regarding how long it will take for the children to arrive
Answer: A. Explain that the client will start to lose consciousness and his body system will
slow down
Rationale:
Expected signs of approaching death include noticeable changes in the client’s level of
consciousness and a slowing down of body systems. The nurse should answer the spouse’s
questions about the signs of imminent death rather than offering reassurance that may or may

not be true. Other options listed may be implemented but the nurse should first answer the
spouse’s question directly.
337. When should intimate partner violence (IPV) screening occur?
A. As soon as the clinician suspects a problem
B. Only when a client presents with an unexplained injury
C. As a routine part of each healthcare encounter
D. Once the clinician confirms a history of abuse
Answer: C. As a routine part of each healthcare encounter
Rationale:
Universal screening for IPV is a vital means to identify victims of abuse in relationship. The
suspicious of different clinicians vary greatly, so screening would not be implemented
consistently. The client should be screened regardless of the presence of injury. Although
history of abuse is difficult to confirm, screening should occur regardless, and this incident
may know may be initial case of abuse.
338. A child newly diagnosed with sickle cell anaemia (SCA) is being discharged from the
hospital. Which information is most important for the nurse to provide the parents prior to
discharge?
A. Instructions about how much fluid the child should drink daily
B. information about non-pharmaceutical pain reliever measures
C. Referral for social services for the child and family
D. Signs of addiction to opioid and medications
Answer: A. Instructions about how much fluid the child should drink daily
Rationale:
It is essential that the child and family understands the importance of adequate hydration in
preventing the stasis-thrombosis-ischemia cycle of a crisis that has a specific plan for
hydration is developed so that a crisis can be delayed. Other choices listed are not the most
important topics to include in the discharge teaching. .
339. What action should the school nurse implement to provide secondary prevention to a
school-age children?
A. Collaborate with a science teacher to prepare a health lesson
B. Prepare a presentation on how to prevent the spread of lice

C. Initiate a hearing and vision screening program for first-graders
D. Observe a person with type 1 diabetes self-administer a dose of insulin
Answer: C. Initiate a hearing and vision screening program for first-graders
Rationale:
Community care occurs at primary, secondary, and tertiary levels of prevention. Primary
prevention involves interventions to reduce the incidence of disease. Secondary prevention
includes screening programs to detect disease. Tertiary prevention provides treatment directed
toward clinically apparent disease. Secondary prevention focuses on screaming children for a
specific disease processes such as hearing and vision screening. The other options are not
examples of secondary prevention.
340. While assisting a client who recently had a hip replacement into a bed pan, the nurse
notices that there is a small amount of bloody drainage on the surgical dressing, the client’s
skin is warm to the touch, and there is a strong Odor from the urine. Which action should the
nurse take?
A. Obtain a urine sample from the bed pan
B. Remove dressing and assess surgical site
C. Insert an indwelling urinary catheter
D. Measure the client’s oral temperature
Answer: D. Measure the client’s oral temperature
Rationale:
The strong Odor from the urine and skin that is warm to the touch may indicate that the client
has a urinary tract infection. Assessing the client’s temperature provides objective
information regarding infection that can be reported to the healthcare provider. Urine should
be obtained via a clean catch, not the bed pan where it has been contaminated. The drainage
on the dressing is normal and does not require direct conservation at this time. An indwelling
catheter should be avoided if possible because it increases the risk of infection.
341. While making rounds, the charge nurse notices that a young adult client with asthma
who was admitted yesterday is sitting on the side of the bed and leaning over the bed-sidetable. The client is currently receiving at 2 litters/minute via nasal cannula. The client is
wheezing and is using pursed-lip breathing. Which intervention should the nurse implement?
A. Assist the client to lie back in bed
B. Call for an Ambu resuscitating bag

C. Increase oxygen to 6 litters/minute
D. Administer a nebulizer Treatment
Answer: D. Administer a nebulizer Treatment
Rationale:
The client needs an immediate medicated nebulizer treatment. Sitting in an upright position
with head and arms resting on the over-bed table is an ideal position to promote breathing
because it promotes lung expansion. Other actions me be accurate but not yet indicated.
342. A client with emphysema is being discharged from the hospital. The nurse enters the
client’s room to complete discharge teaching. The client reports feeling a little short of breath
and is anxious about going home. What is the best course of action?
A. Postpone discharge instructions at this time and offer to contact the client by phone in a
few days
B. Invite the client to return to the unit for discharge teaching in a few days, when there is
less anxiety
C. Provide only necessary information in short, simple explanations with written instructions
to take home
D. Give detailed instructions speaking slowly and clearly while looking directly at the client
when speaking
Answer: C. Provide only necessary information in short, simple explanations with written
instructions to take home
Rationale:
Simple, short explanations should be provided. Information is not retained when the recipient
is anxious, and too much information can increase worry. Ethically, discharge instructions
may not be postponed.
343. An older adult male who had an abdominal cholecystectomy has become increasingly
confused and disoriented over the past 24 hours. He is found wandering into another client’s
room and is return to his room by the unlicensed assistive personnel (UAP). What actions
should the nurse take? (Select all that apply).
A. Apply soft upper limb restrains and raise all four bed rails
B. Report mental status change to the healthcare provider
C. Assess the client’s breath sounds and oxygen saturation
D. Assign the UAP to re-assess the client’s risk for falls

E. Review the client’s most recent serum electrolyte values
Answer: B. Report mental status change to the healthcare provider
C. Assess the client’s breath sounds and oxygen saturation
E. Review the client’s most recent serum electrolyte values
Rationale:
The healthcare provider should be informed of changes in the client’s condition (B) because
this behavior may indicate a postoperative complication. Diminished oxygenation (C) and
electrolyte imbalance (E) may cause increased confusion in the older adult. Raising all four
bed rails (A) may lead to further injury if the client climbs over the rails and falls and
restrains should not be applied until other measures such as re-orientation are implemented.
The nurse should assess the client’s increased risk for falls, rather than assigning this to the
UAP (D).
344. A client is admitted to a medical unit with the diagnosis of gastritis and chronic heavy
alcohol abuse. What should the nurse administered to prevent the development of Wernicke's
syndrome?
A. Lorazepam (Ativan)
B. Famotidine (Pepcid)
C. Thiamine (Vitamin B1)
D. Atenolol (Tenormin)
Answer: C. Thiamine (Vitamin B1)
Rationale:
Thiamine replacement is critical in preventing the onset of Wernickes encephalopathy, an
acute triad of confusion, ataxia, and abnormal extraocular movements, such as nystagmus
related to excessive alcohol abuse. Other medications are not indicated.
345. When conducting diet teaching for a client who was diagnosed with nutritional anaemia
in pregnancy, which foods should the nurse encourage the client to eat? (Select all that apply)
A. Seeds, spices, lettuce
B. Consomme, celery, carrot
C. Oranges, orange juice, bananas
D. Fortified whole wheat cereals, whole-grain pasta, brown rice
E. Spinach, kale, dried raisins and apricots
Answer: D. Fortified whole wheat cereals, whole-grain pasta, brown rice

E. Spinach, kale, dried raisins and apricots
Rationale:
Nutritional anaemia in pregnancy should be supplemented with additional iron in the diet.
Foods that are high in iron content are often protein based, whole grains (D), green leafy
vegetables and dried fruits (E). (A, B, and C) are not iron rich sources
346. A client with type 2 diabetes mellitus is admitted for antibiotic treatment for a leg ulcer.
To monitor the client for the onset of hyperosmolar hyperglycaemic nonketotic syndrome
(HHNS), what actions should the nurse take? (Select all that apply)
A. Check urine for ketones
B. Measure blood glucose
C. Monitor vital signs
D. Assessed level of consciousness
E. Obtain culture of wound
Answer: B. Measure blood glucose
C. Monitor vital signs
D. Assessed level of consciousness
Rationale:
Blood glucose greater than 600 mg/dl (33.3 mmol/L SI), vital sign changes in mental
awareness are indicators of possible HHNS. Urine ketones are monitored in diabetic
ketoacidosis. Wound culture is performed prior to treating the wound infection but is not
useful in monitoring for HHNS.
347. An infant is receiving penicillin G procaine 220,000 units IM. The drug is supplied as
600,000 units/ml. How many ml should the nurse administer? (Enter numeric value only. If
rounding is required, round to the nearest tenth)
Answer: 0.4
Rationale:
Calculate using the formula, desired dose (220,000 units) over dose on hand (600,000 units) x
the volume of the available dose (1 ml). 220,000 / 600,000 x 1 ml = 0.36 = 0.4 ml
348. After receiving report, the nurse can most safely plan to assess which client last? The
client with…
A. A rectal tube draining clear, pale red liquid drainage

B. A distended abdomen and no drainage from the nasogastric tube
C. No postoperative drainage in the Jackson-Pratt drain with the bulb compressed
D. Dark red drainage on a postoperative dressing, but no drainage in the Hemovac®.
Answer: D. Dark red drainage on a postoperative dressing, but no drainage in the
Hemovac®.
Rationale:
The most stable client is the one with a functioning drainage device and no drainage. This
client can most safely be assesses last. Other clients are either actively bleeding, have an
obstruction in the nasogastric tube which may result in vomiting, or may be bleeding and / or
may have a malfunction in the Hemovac® drain.
349. The nurse instructs an unlicensed assistive personnel (UAP) to turn an immobilized
elderly client with an indwelling urinary catheter every two hours. What additional action
should the nurse instruct the UAP to take each time the client is turned?
A. Empty the urinary drainage bag
B. Feed the client a snack
C. Offer the client oral fluids
D. Assess the breath sounds
Answer: C. Offer the client oral fluids
Rationale:
Increasing oral fluid intake reduces the risk of problems associated with immobility, so the
UAP should be instructed to offer the client oral fluids every two hours, or whenever turning
he client. It is not necessary to empty the urinary bag or feed the client every two hours.
Assessment is a nursing function, and UAPs do not have the expertise to perform assessment
of breath sounds.
350. The nurse is preparing a client who had a below-the-knee (BKA) amputation for
discharge to home. Which recommendations should the nurse provide this client? (Select all
that apply)
A. Inspect skin for redness
B. Use a residual limb shrinker
C. Apply alcohol to the stump after bathing
D. Wash the stump with soap and water
E. Avoid range of motion exercises

Answer: A. Inspect skin for redness
B. Use a residual limb shrinker
D. Wash the stump with soap and water
Rationale:
Several actions are recommended for home care following an amputation. The skin should be
inspected regularly for abnormalities such as redness, blistering, or abrasions. A residual limb
shrinker should be applied over the stump to protect it and reduce edema. The stump should
be washed daily with a mild soap and carefully rinse and dried. The client should avoid
cleansing with alcohol because it can dry and crack the skin. Range of motion should be done
daily.
351. When assessing the surgical dressing of a client who had abdominal surgery the previous
day, the nurse observes that a small amount of drainage is present on the dressing and the
wound’s Hemovac suction device is empty with the plug open. How should the nurse
respond?
A. Replace the dressing and remove the drainage device
B. Reposition the drainage device and keep the plug open
C. Notify the healthcare provider that the drain is not working
D. Recompress the wound suction device and secure to plug
Answer: D. Recompress the wound suction device and secure to plug
Rationale:
The plug of a wound suction device, such as a Hemovac, should be closed after compressing
the device to apply gentle suction in a closed surgical wound to facilitate the evacuation of
subcutaneous fluids into the device. Compressing the device and securing the plug should
restore function of the closed wound device. A small amount of drainage should be marked
on the dressing, but replacing the dressing is not necessary and the nurse should not remove
the device. Other options are not indicated.
352. A mother brings her 4-month-old son to the clinic with a quarter taped over his
umbilicus, and tells the nurse the quarter is supposed to fix her child’s hernia. Which
explanations should the nurse provide?
A. This hernia is a normal variation that resolves without treatment.
B. Restrictive clothing will be adequate to help the hernia go away.
C. An abdominal binder can be worn daily to reduce the protrusion.

D. The quarter should be secured with an elastic bandage wrap.
Answer: A. This hernia is a normal variation that resolves without treatment.
Rationale:
An umbilical hernia is a normal variation in infants that occurs due to an incomplete fusion of
the abdominal musculature through the umbilical ring that usually resolves spontaneously as
the child learns to walk. Other choices are ineffective and unnecessary.
353. A client who is admitted to the intensive care unit with syndrome of inappropriate
antidiuretic hormone (SIADH) has developed osmotic demyelination. Which intervention
should the nurse implement first?
A. Patch one eye.
B. Reorient often.
C. Range of motion.
D. Evaluate swallow
Answer: D. Evaluate swallow
Rationale:
Osmotic demyelination, also known as central pontine myelinolysis, is nerve damage caused
by the destruction of the myelin sheath covering nerve cells in the brainstem. The most
common cause is a rapid, drastic change in sodium levels when a client is being treated for
hyponatremia, a common occurrence in SIADH. Difficulty swallowing due to brainstem
nerve damage should be care, but determining the client’s risk for aspiration is most
important.
354. A client with possible acute kidney injury (AKI) is admitted to the hospital and mannitol
is prescribed as a fluid challenge. Prior to carrying out this prescription, what intervention
should the nurse implement?
A. Collect a clean catch urine specimen.
B. Instruct the client to empty the bladder.
C. Obtain vital signs and breath sounds.
D. No specific nursing action is required
Answer: C. Obtain vital signs and breath sounds.
Rationale:
The client’s baseline cardiovascular status should be determined before conducting the fluid
challenge. If the client manifests changes in the vital signs and breath sounds associated with

pulmonary edema, the administration of the fluid challenge should be terminate. Other
options would not assure a safe administration of the medication.
355. A male client with COPD smokes two packs of cigarettes per day and is admitted to the
hospital for a respiratory infection. He complains that he has trouble controlling respiratory
distress at home when using his rescue inhaler. Which comment from the client indicates to
the nurse that he is not using his inhaler properly?
A. “I have a hard time inhaling and holding my breath after I squeeze the inhaler, but I do my
best”
B. “ I never use the inhaler unless I am feeling really short of breath”
C. I always shake the inhaler several times before I start”
D. “After I squeeze the inhaler and swallow, I always feel a slight wave of nausea, bit it goes
away”
Answer: D. “After I squeeze the inhaler and swallow, I always feel a slight wave of nausea,
bit it goes away”
356. A nurse is planning to teach infant care and preventive measures for sudden infant death
syndrome (SIDS) to a group of new parents. What information is most important for the
nurse to include?
Answer: Ensure that the infant’s crib mattress is firm
357. A 6 -years-old who has asthma is demonstrating a prolonged expiratory phase and
wheezing, and has 35% personal best peak expiratory flow rate (PEFR). Based on these
finding, which action should the nurse implement first?
A. Administer a prescribed bronchodilator.
B. Report finding to the healthcare provider.
C. Encourage the child to cough and deep breath
D. Determine what trigger precipitated this attack.
Answer: A. Administer a prescribed bronchodilator.
Rationale:
If the PEFR is below 50% in as asthmatic child, there is severe narrowing of the airway, and a
bronchodilator should be administered immediately. Be should be implemented after A. C
will not alleviate the symptoms and D is not a priority.

358. A client is receiving lactulose (Portalac) for signs of hepatic encephalopathy. To evaluate
the client’s therapeutic response to this medication, which assessment should the nurse
obtain?
A. Level of consciousness
B. Percussion of abdomen
C. Serum electrolytes
D. Blood glucose.
Answer: A. Level of consciousness
Rationale:
Colonic bacteria digest lactulose to create a drug-induces acidic and hyperosmotic
environment that draws water and blood ammonia into the colon and coverts ammonia to
ammonium, which is trapped in the intestines and cannot be reabsorbed into the systemic
circulation. This therapeutic action of lactulose is to reduce serum ammonia levels, which
improves the client’s level of consciousness and metal status.
359. When administering an immunization in an adult client, the nurse palpates and
administer the injection one inch below the acromion process into the center of the muscle
mass. The nurse should document that the vaccine was administered at what site?
A. Rectus femenis
B. Ventrogluteous
C. Vastus lateralis
D. Deltoid
Answer: D. Deltoid
Rationale:
The acromion process is a parameter identified for the deltoid site.
360. A primigravida a 40-weeks’ gestation with preeclampsia is admitted after having a
seizure in the hot tub at a midwife’s birthing centre. Based on documentation in the medical
record, which action should the nurse implement? (Click on each chart tab for additional
information. Please be sure to scroll to the bottom right corner of each tab to view all
information contained in the client’s medical record.)
Answer: Continue to monitor the client’s blood pressure hourly

Document Details

  • Subject: Nursing
  • Exam Authority: HESI
  • Semester/Year: 2023

Related Documents

person
Charlotte Scott View profile
Close

Send listing report

highlight_off

You already reported this listing

The report is private and won't be shared with the owner

rotate_right
Close
rotate_right
Close

Send Message

image
Close

My favorites

image
Close

Application Form

image
Notifications visibility rotate_right Clear all Close close
image
image
arrow_left
arrow_right